You are on page 1of 92

TORTS 2

CHAPTER 8 - DAMAGES ................................................................................2


INTRODUCTION.....................................................................................................2
PERSONAL INJURY.................................................................................................4
RODRIGUEZ v. MCDONNELL DOUGLAS CORP...........................................................................4
MCDOUGALD v. GARBER...........................................................................................................6
HAYNES v. YALENEW HAVEN HOSPITAL.................................................................................8
WRONGFUL DEATH AND SURVIVAL ACTIONS.........................................................11
ASPINALL v. MCDONNELL DOUGLAS CORP.............................................................................13
McDAVID v. UNITED STATES...................................................................................................14
OTHER COMPENSATORY DAMAGE ISSUES.............................................................16
PUNITIVE DAMAGES ............................................................................................17
STATE FARM MUTUAL AUTO. INS. CO. v. CAMPBELL...............................................................17
CHAPTER 9- AFFIRMATIVE DEFENSES BASED ON PLAINTIFFS CONDUCT ........21
INTRODUCTION...................................................................................................21
CONTRIBUTORY AND COMPARATIVE NEGLIGENCE .................................................23
HOFFMAN v. JONES ................................................................................................................23
WASSELL v. ADAMS................................................................................................................25
FAILURE TO AVOID CONSEQUENCES; FAILURE TO MITIGATE DAMAGES ...................26
DARE v. SOBULE.....................................................................................................................26
HUTCHINS v. SCHWARTZ.......................................................................................................28
ASSUMPTION OF THE RISK ..................................................................................30
BENNETT v. HIDDEN VALLEY GOLF AND SKI, INC....................................................................30
IMPUTED CONTRIBUTORY FAULT .........................................................................32
CONTINENTAL AUTO LEASE CORP. v. CAMPBELL....................................................................32
WHITE v. LUNDER...................................................................................................................34
CHAPTER 10- OTHER AFFIRMATIVE DEFENSES..............................................36
STATUTES OF LIMITATIONS..................................................................................36
JOLLY V. ELY LILLY & CO..........................................................................................................36
FELTMEIER v. FELTMEIER........................................................................................................38
STATUTES OF REPOSE ........................................................................................40
BRADWAY v. AMERICAN NATIONAL RED CROSS.....................................................................40
EXPRESS ASSUMPTION OF RISK ..........................................................................42
HOLZER v. DAKOTA SPEEDWAY, INC. .....................................................................................42
CHAPTER 11 COMPARATIVE RESPONSIBILITY IN MULTIPARTY LITIGATION. . . .44
CONTRIBUTION AND INDEMNITY ..........................................................................44
BROCHNER v. WESTERN INSURANCE COMPANY.....................................................................44
JOINT AND SEVERAL LIABILITY ............................................................................46
KAEO v. DAVIS........................................................................................................................46
BROWN v. KEILL .....................................................................................................................48
VARELA v. AMERICAN PETROFINA COMPANY OF TEXAS, INC..................................................49
Partial Settlements ............................................................................................51
MCDERMOTT, INC. v. AMCLYDE ..............................................................................................51
APPLICATION OF COMPARATIVE RESPONSIBILITY TO CAUSES OF ACTION OTHER
THAN NEGLIGENCE..............................................................................................54

1
BONPUA v. FAGAN..................................................................................................................54
CHAPTER 12 - IMMUNITIES .........................................................................56
A. GOVERNMENT IMMUNITY ..............................................................................56
1. STATE AND LOCAL GOVERNMENT ....................................................................................56
HICKS v. STATE..................................................................................................................................56
2. FEDERAL GOVERNMENT ...................................................................................................61
a. FEDERAL TORT CLAIMS ACT ........................................................................................................61
UNITED STATES v. GAUBERT .........................................................................................................61
b. JUDICIALLY CREATED IMMUNITY .................................................................................................62
UNITED STATES v. JOHNSON .........................................................................................................62
3. IMMUNITY OF GOVERNMENTAL OFFICERS AND EMPLOYEES.............................................64
4. THE PUBLIC DUTY DOCTRINE ........................................................................................64
RISS v. CITY OF NEW YORK................................................................................................................64
B. FAMILY IMMUNITIES......................................................................................66
PRICE v. PRICE........................................................................................................................66
BROADBENT v. BROADBENT...................................................................................................67
CHAPTER 13 - MEDICAL MALPRACTICE..........................................................69
PROFESSIONAL STANDARD OF CARE....................................................................69
MELVILLE v. SOUTHWARD.......................................................................................................69
HARNISH v. CHILDRENS HOSPITAL MEDICAL CENTER............................................................72
CHAPTER 14 COMMON LAW STRICT LIABILITY.............................................75
ABNORMALLY DANGEROUS ACTIVITIES.................................................................75
RYLANDS v. FLETCHER............................................................................................................75
SEIGLER v. KUHLMAN..............................................................................................................78
CHAPTER 15 - PRODUCTS LIABILITY.............................................................80
EMERGENCE OF STRICT TORT LIABILITY ...............................................................80
GREENMAN v. YUBA POWER PRODUCTS, INC.........................................................................80
DEFECT ..............................................................................................................81
GRAY v. MANITOWOC CO., INC................................................................................................81
PHILLIPS v. KIMWOOD MACHINE CO........................................................................................82
FELDMAN v. LEDERLE LABORATORIES....................................................................................85
POTTER v. CHICAGO PNEUMATIC TOOL CO.............................................................................87

CHAPTER 8 - DAMAGES
INTRODUCTION
A. Categories of Damages:
2
- There are 3 main categories of damages: nominal damages, compensatory damages, and
punitive damages

- The cause of actions for some intention torts is complete without proof of actual damages.
When a plaintiff wins such a case without proving any damages, the court awards
nominal damages, which may be defined as a trivial sum of money awarded to a
litigant who has established a cause of action but has not established that he is entitled to
compensatory damages.- Restatement 2d. Designed to vindicate a right.

- Compensatory Damages- the damages awarded to a person as compensation,


indemnity, or restitution for harm sustained by him.- Restatement 2d. The basic theory
behind compensatory damages is restoration of the plaintiff to his or her pre-injury
condition, to the extent that an award of money can do that. In causes of action based on
strict liability and negligence, the plaintiff must prove compensatory damages as a part of
the prima facie case. Most of the litigation occurs under compensatory damages.

- Punitive Damages- the damages, other than compensatory or nominal damages,


awarded against a person to punish him for his outrageous conduct and to deter him and
other like him from similar conduct in the future- Restatement 2d. To punish and deter
the wrongdoer from heinous conduct (some type of outrageous damages)

B. Compensatory Damages For Physical Harms to Property

- For an invasion of land, the basic measure of recovery is any diminution in the lands
value caused by the invasion, or at the plaintiffs election, the reasonable costs of
restoration, plus the value of the plaintiffs loss of use of the land (the rental value.)

- When a chattel has been converted or destroyed, the basic measure of damages is its
entire value at the time and place of the tort (fair market value).

- When a chattel has been damaged, but not destroyed, the basic measure of damage
is the diminution in value caused by the tort (repair costs are often relevant, but are not
the final measure).

- In all of these cases, the court may also award consequential or incidental damages.

o Business profits (if proved with sufficient certainty)

o Example of trespass to land including damages for injury to person or chattels


caused by the trespasser.

C. Compensatory Damages In Personal Injury Cases

- Three main types of personal injury compensatory damages:

1. Medical (and related) expenses (past and future)- pecuniary damages/ special
damages because they are capable of economic evaluation, even though future
losses are uncertain. Past medical expenses are established by the hospital,
doctor bills. Future losses generally require expert testimony to establish what
kinds of expenses are likely to be incurred and how much the expenses are likely
to be.

2. Loss of earning capacity (past and future)- includes loss of past earnings
pecuniary damages/special damages . Typically past loss of earnings are
established by projecting what the plaintiff was earning at the time of the injury
through the date of the trial. However, this may require expert testimony if there
would have been a change in circumstances, such as promotion, industry layoffs,
etc. Future earning capacity loss is typically proven through the use of experts.
3
These may include a variety of types of experts including economists, labor
specialists, rehabilitation experts, etc.

3. Physical and mental pain and suffering (past and future)- non-pecuniary
damages, referring to pain and suffering that cant be easily measured in dollars.
This is where juries have the most leeway because there really isnt anything
to follow. These are the type of damages that are sometimes known as general
damages, are those not susceptible of economic evaluation. A variety of
evidence is used to give the jury a basis for making a finding on the issue of
general damages.

a. Wide variety of evidence-

i. Plaintiff testimony

ii. Family members

iii. Doctors, therapists, counselors, are often part of Ps case

iv. Photographs

v. Videos

vi. Charts and diagrams

vii. Computer graphics

viii. Thermographic imaging- supposed to show areas of pain according


to heat/inflammation

PERSONAL INJURY

RODRIGUEZ v. MCDONNELL DOUGLAS CORP


California Court of Appeals, 1978

87 Cal.App.3d 626, 151 CalRptr. 399

Page 328 in casebook

Procedural History- At the conclusion of the trial in 1975, McDonnell Douglas was exonerated
but found both of the contractors liable for damages to Rodriguez and his wife in the value of
over 4.5 million dollars.

Facts- Plaintiff, Richard Rodriguez, was a sprinkler fitters apprentice for a fire protection
company that subcontracted to McDonnell Douglas for a hangar modification project. Due to
the negligence of two other contractors, a 630 pound pipe fell on Rodriguez, causing him to
become a triplegic. He would never be able to walk again. He suffered massive brain damage,
a fractured spine and irreparable damage to his spinal cord. He was subsequently
hospitalized many times and had many surgeries as well. Also lost kidney, bowl, and bladder
functions.

4
Issue(s)- Did the trial court err in overruling the defendants motion for a new trial on the
ground that the size of the award suggested that the jury acted on passion and prejudice?

Holding- No. The awards to the plaintiff were reasonable given the grievous nature of the
injuries Rodriguez received. The defendants produced no evidence that the awards the jury
gave reflected passion and prejudice. The award did not shock the conscience.

Rule(s)- The determination of damages is primarily a factual matter on which the inevitable
wide differences of opinion do not call for the intervention of appellate courts. An appellate
court, in reviewing the amount of damages, must determine every conflict in the evidence in
respondents favor and give him the benefit of every reasonable inference. An appellate court
may not interfere with an award unless the verdict is so large that, at first blush, it shocks
the conscience and suggests passion, prejudice, or corruption on the part of the jury.

Analysis-

Disposition- Affirmed

Notes from Class-

- What rule did the court apply in reviewing the damage award to the plaintiff?

o Appellate may not interfere with the size of the jurys award unless it is so large that
it shocks the conscience and suggests passion, prejudice or corruption

- Relatively rare that a judge will overturn an award

- The jury is not told what other jurys award other people so there is no going rate
when it comes to giving awards to plaintiffs (especially when it comes to pain and
suffering, there is no real rule)

- An award could shock the conscience if it is too high or if it is too low. There is a lot of
strategizing for where the case is filed depending on the community you are in (juries are
different)

- How does one establish loss of earning capacity?

o Generally an expert witness that is an economist

o The younger the person is who has a long work history, then the award is going to
be less

- Earning Capacity

o What is the nature of the loss?

What the person could have earned

- How does the D deal with testimony of the Ps economist?

5
o Sometimes the best defense is not to say anything, to keep him from talking, unless
you can undermine their data, or inconsistencies in their facts

- What is the obligation of spouse with respect to attendant care?

o There is not one as a wife to render 24 hours in a day assistance. This would be
unreasonable. She shouldnt have to require care around the clock.

- Inflation

o This is taken into account because over time inflation changes and things are not
worth the same value as they were before.

- Discount Rate

o Compounding or calculation of a future value

o 336-337

o Economist picks out the appropriate discount rate based on his experience and
education

- Present value

- Consortium

o Majority view- In a non death claim, the spouse of the injured party may have a
claim for loss of consortium (loss of sexual relationship, companionship, love and
other services

o Generally applies to spouses

o Minority view-Some states allow children to recover for loss of society,


companionship, or guidance in non-fatal cases and some allow parents to recover
loss of consortium for injuries to children

- Reducing awards

o New trial (complete or partial)

Remittur the award was excessive, reduce the award, can terminate the
case and accept the reduced award or it can be remanded for a new trial

Additur- the award was too little, the jury didnt award enough, allows the
option to bring the award up to a certain amount of money. The defendant
has the option to go back to trial or not.

- Present value- pecuniary damages are generally reduced to present value. However, non-
pecuniary damages are not.

MCDOUGALD v. GARBER
New York Court of Appeals, 1989

73 N.Y.2d 246, 538 N.Y.S.2d 937, 536

Page 337 in casebook

6
Procedural History- Action was brought by McDougald and her husband against the
defendants for malpractice. Jury awarded Emma McDougald $9,650,102 in damages for
conscious pain and suffering and for the loss of the pleasures and pursuits of life. Her
husband was awarded $1.5 million for the loss of his wifes services. The trial judge reduced
her reward to $4,796,728 by striking the entire award for future nursing care and reducing
the award for conscious pain and suffering and for the loss of the pleasures and pursuits of
life. On cross appeal, the appellate division affirmed and later granted defendants leave to
appeal to the N.Y. Court of Appeals.

Facts- Plaintiff, Emma McDougald, underwent a C-section and tubal ligation at N.Y. Infirmary.
Defendant, Garber, performed the surgery and defendants, Armengol and Kulkarni, provided
the anesthesia. During the surgery, McDougald suffered oxygen deprivation which resulted in
severe brain damage and left her in a permanent comatose condition.

Issue(s)- Did the trial court err in instructing the jury that Ps awareness was irrelevant to
their consideration of damages for loss of enjoyment of life and in directing the jury to
consider that aspect of damages separately from pain and suffering? Does the P have to have
some level of awareness? Should loss of enjoyment of life be considered a category of
damages separate from pain and suffering?

Holding- Yes the trial court erred in instructing the jury that Ps awarenessThere must be
some cognitive awareness in order to recover nonpecuniary damages for the loss of
enjoyment of life. However, the fact finder does not have to determine degrees of cognitive
awareness in order for a plaintiff to recover. In addition, just because enjoyment of life and
pain and suffering can be distinguished, the two do not need to be made into separate
categories.

Rule(s)-

1. Cognitive awareness is a prerequisite to recovery for loss of enjoyment of life. There must
be some level of awareness in order for a plaintiff to recover.

2. Punitive damages, those which have no compensatory purpose, are prohibited unless the
harmful conduct is intentional, malicious, outrageous, or otherwise aggravated beyond
mere negligence.

Analysis- Jury should not make separate awards for pain and suffering and loss of enjoyment of
life.

Some states do hold that a P need not be conscious to recover damages for loss of
enjoyment of life (Hedonic Damages). Some states also permit a separate award for
loss of enjoyment in addition to the award for pain and suffering.

Disposition- Remanded for a new trial on the issue of nonpecuniary damages for Emma
McDougald

7
Notes from Class

- Compensatory purpose v. punitive purpose- compensation v. punishment/deterrence

- Does the injured victim (P) have to be cognitively aware of pain and suffering in order to
recover damages for such pain and suffering?

o Must be some conscious awareness

o Not an exact level of consciousness, however, there must be some awareness of


pain and suffering

- Does the plaintiff have to be aware of the loss of pleasures and pursuits of life (hedonic
damages)?

o This Court said yes you have to be aware that you have lost something

o No standard of awareness though

- Should the loss of enjoyment of life be treated as a separate item of damages from pain
and suffering?

o No, these are really one item. It would distort the amount of the award and possibly
bump it up. Generally, courts do not treat them as separate line items.

o Not a damage that is measurable in economic terms. It is all about what the loss
means to the plaintiff. If they are not aware of the loss, then it does not mean
anything to them.

- Disagreement among states:

o Some allow recovery for loss of enjoyment (hedonic) damages whether or not the P
is aware of the loss

- A second issue is whether pain and suffering should be treated separately from hedonic
damages in making an award

o States are split on this

- Not all legal jurisdictions make general awards for pain and suffering. Other places like
Europe do not give out these awards like America does.

- Notes 4 and 5

HAYNES v. YALENEW HAVEN HOSPITAL


Supreme Court of Connecticut, 1997

243 Conn. 17, 699 A.2d 964

Page 345 in Casebook

8
Procedural History- Decedents daughter, Susan Haynes, brought a wrongful death suit
against Perrier, the other driver, and received a settlement of $20K from his insurance
agency. She then received $630K from her own insurance agency. She then filed a
malpractice claim against the hospital and the surgeon. The trial court granted the
defendants motion for summary judgment on the ground that the plaintiff had already
received full compensation for the harm suffered by her decedent.

Facts- Barbara Freeman, plaintiffs decedent, was injured when she was in a car accident with
Alan Perrier. She was taken to Yale-New Havens emergency room and was treated for a
fractured left leg and pelvis. After an hour and a half, the emergency room doctors noticed
she was experiencing an expanding abdominal girth. She was taken into surgery, and the
doctor noticed large amounts of blood in her abdomen due to a laceration of the spleen.
During the surgery she went into cardiac arrest and died. She was awarded $20,000, which
was the limit of his insurance coverage. Freeman had purchased an additional $900,000
worth of coverage for herself. An arbitration panel fixed the total damages at $650,000 and
the insurance company paid Haynes $630,000 after deducting $20,000 she had received
from Perriers insurance carrier.

Issue(s)- Did the trial court err in granting the defendants motion for summary judgment? Do
underinsured motorists benefits fall within the common law rule precluding double recovery
or do they fall within the common law collateral source rule?

Holding- No. The fundamental principle behind the underinsured motorist insurance is to
place the insured in the same position as, but no better than, the insured would have been
had the underinsured tortfeasor been fully insured. The Ps argument would allow her to
recover MORE than her full damages.

Rule(s)- Double-recovery is precluded by policy. We are trying to place the injured party in
the same position he/she would have been had the accident not occurred.

Analysis-

Disposition- Affirmed

Notes-

Haynes v. Yale-New Haven Hospital

- Plaintiffs mother died in the emergency room following a car wreck caused by another
motorist

- P settled a wrongful death claim against the motorist, receiving 20K, the limits of the
motorists liability insurance policy.
9
- P then brought a claim against the hospital and the surgeon alleging malpractice. Then Ds
moved for summary judgment on the grounds that plaintiff had already received full
compensation of 650k. Motion was granted.

- Issue: Did the trial court err in finding that the insurance payment from the mothers
insurer under

- In other words, should the underinsured motorist coverage be treaes under the double
recovery or the collateral source rule?

- Double recovery rule

o You can only recover one time

- Collateral Source Rule:

o A D is not entitled to a credit against the damage award to P for those amounts

- The court in this case said that the payments by the underinsured motorist coverage
insurer

- More courts apply the collateral source rule than the double recovery rule to underinsured
motorist insurance

- Subrogation:

o The right to claim recoupment from proceeds paid to the P from the tortfeasor or
the tortfeasors insurer for payments to the P by Ps insurer (or employers insurer)

o This may in fact result in the right of the subrogated party to sue the tortfeasor
directly or to intervene in a lawsuit brought by the injured party to protect the
subrogation rights.

o May be granted by statute (typically the workers compensation insurer is


subrogated by statute to the injured workers claim against 3rd persons), or by
contract (typically the health insurer is subrogated by contract)

o Example:

If a workers comp insurer pays benefits such as medical expenses on behalf


of an injured worker, the workers comp insurer

o Typically if a settlement is involved the WC insurer must be involved in the


settlement with the tortfeasor.

o Subrogation is also typical in insurance agreements

- Insurance issues:

o Who gets stuck with the loss between the insurers?

- Collateral benefits- received by injured parties from sources other than D, such as medical
or car insurance. It is something that the Plaintiff will have paid for.

- If you are only entitled to 100k from D, but you have insurance that will pay 50k. How
much can you recover? Collateral benefit says that the insurance doesnt count toward the
100k that D owes.
10
- TORT REFORM

WRONGFUL DEATH AND SURVIVAL ACTIONS


- Common Law permitted no recovery for wrongful death actions. The explanation was that
fell under the felony-merger rule, premised on the notion that the negligent or intentional
killing of another was a felony, punishable by death. Thus there was no defendant to sue.
Another explanation was that the law generally does not give one person a cause of action
for a tort committed against another.

- Mid-19th century, American jurisdictions adopted statutes permitting actions for wrongful
death. The statutes specified who could recover (usually spouse, children, sometimes
parents and siblings, sometimes all the decedents legal heirs), and who could bring suit
(usually the person authorized to administer the decedents estate, sometimes any of the
persons authorized to recover. (Statutorily created with designated beneficiaries)

- Wrongful death action and survival actions are both statutorily created

- Beneficiaries (varies from state to state), everything is statute provided

o Surviving Spouse is always included

o Almost all if not all include the children

o Parents and Siblings may be able to recover in the absence of spouse or children
(generally speaking, they are last in line)

o Sometimes other heirs and collateral relatives (out of the immediate family)

o Estate or legatees

- Wrongful death), everything is statute provided

o Can recover for:

Loss of support

Loss of companionship

Loss of guidance

Loss of services

o Some states allow for grief and mental anguish (but not in most states)

o Cause of action exists when the deceased could have brought a claim for his own
injuries. When there has been a tort committed against the deceased for which he
could have sought recovery if he had survived, then the wrongful death action
would apply.

o Typically the estate rep (the executor of the will or the administrator) must bring
the suit. In some states, the beneficiaries can bring suit in their own names.

o If various beneficiaries bring suit, typically all beneficiaries must adjudicate their
claims in the same suit.

11
o The statute may specifically provide how the proceeds recovered are to be divided
among the beneficiaries.

- Survival Statutes

o The claim that the deceased could have brought on her own behalf and then she
dies, but she could have filed before she died, then that claim survives

o Allows the claims of deceased that came into existence before death to proceed.
Some states require that the action must be filed before death. Others only require
the cause of action to have come into being, but do not require filing before death,
as long as it falls within the SOL.

o Claims that arose against the decedent before death may be prosecuted against the
estate (through the estate representative).

o Damages would typically include:

Post-injury, pre-death pain and suffering of the decedent

Loss of earnings from the time of injury until the time of death

Medical and incidental expenses

Funeral expenses

Property damage claims

o A number of states do not permit the survival of defamation claims

o Punitive damage claims typically do not survive defendants death

o Some states do not permit punitive damage awards when the injured person dies
before the judgment

o Medical and funeral expense claims may depend on who pays them or owes them.
Sometimes they are recoverable in the wrongful death action.

o A few states allow the recovery of loss of earning capacity in survival actions (i.e.,
earning lost because of the death itself), but subtract personal expenses and
support to others (or allow recovery for loss of net savings)

o Some states have statutes that in essence merge both wrongful death and survival
claims into one action

o Majority- both actions may be brought together

- Punitive damages:

o Purpose:

To punish (retribution)

To deter (defendant and others)

o Punitive damages may be awarded when the defendants conduct may be regarded
as willful or wanton, or where the defendant acts with malice or reckless disregard
of the safety of others.
12
o An intentional tort doesnt necessarily mean there will be punitive damages. I.e. a
good faith trespass. You must prove the intent.

o Standard of proof:

Preponderance of evidence

Clear and convincing evidence (Mississippi standard)

It is not beyond a reasonable doubt

Requires more than it is more likely than not

Must be obvious

Between beyond reasonable doubt and more likely than not

Must be convincing to the jury, cant be a he probably did it

Greater than 51%

o Punitive damages award may be considered as part of the principal case or it may
be considered separately after any compensatory damage award is made.

o May state not have caps on punitive damages

o Some states require that a significant portion of the punitive damages go to the
state rather than the plaintiff on the theory that since they are acting on behalf of
the public, it should benefit the public in the state and not only the plaintiff.

ASPINALL v. MCDONNELL DOUGLAS CORP


United States Court of Appeal, Ninth Circuit, 1980

625 F.2d 325

Procedural History- Appellant Aspinall, as representative of the estate of Anthony Price,


appeals from the district courts order for summary judgment holding that she and her
children are not heirs to his estate for the purposes of the California Wrongful Death Act.

Facts- Anthony Price was killed in a plane crash that was manufactured by McDonnell
Douglas. At the time of his death, he was unmarried and had no issue. His parents were
deceased and he had no collateral heirs. However, he and the appellant lived together for
over 4 years and the decedent left his estate to her in his will. Price supported the
appellant and her children , but never married Aspinall and never adopted her children.

Issue(s)- Did the district court err in holding that Aspinall and her children were not heirs
to the Price estate under the California Wrongful Death Act?

13
Holding- No. The act requires that in order for Aspinall to have been an heir, she and Price
would have had to have been married. In order for her children to have been heirs, Price
would have had to have adopted them. Because he neither married Aspinall nor adopted
her children, Aspinall and her children cannot be considered heirs under the California
Wrongful Death Act.

Rule(s)- Section 377 of the California Wrongful Death Act defined heirs as those who
would have been eligible to inherit from the decedents estate had he died intestate- i.e.
having no valid will/last testament before death.

Analysis-

Disposition- Affirmed

Wrongful death for the survivorship (those suing on behalf of the decedent) to recover

Survival actions actions for recovery brought prior to the death of the plaintiff

McDAVID v. UNITED STATES


Supreme Court of Appeals of West Virginia, 2003.

Page 353

Procedural History- P sued in federal court, alleging negligence by govt. physicians that caused
her husbands death. She also sought damages for his emotional distress and pain and suffering
from the time of negligence until his death, and for his loss of enjoyment of life. Government
moved to dismiss the claim for pain and suffering.

Certified question given: Whether a decedents beneficiaries may recover damages for a
decedents pain and suffering, incurred between the time of injury and the time of death, where
the injuries result in death but the decedent did not institute an action for personal injury prior to
his death?

Facts- Ps husband died while being treated at a VA hospital.

Issue(s)- Whether a decedents beneficiaries may recover damages for a decedents pain and
suffering, incurred between the time of injury and the time of death, where the injuries result in
death but the decedent did not institute an action for personal injury prior to his death?

Holding- Under the wrongful death act, W.Va.Code 55-7-6, a jurys verdict may include damages
for the decedents pain and suffering endured between the time of injury and the time of death,
14
where the injury resulted in death but the decedent did not institute an action for personal injury
prior to his or her death.

Rule(s)- 1. Loss to Survivors Theory- Majority Rule- Under these statutes, damages generally
are to be assessed in accordance with the loss to the decedents survivors, with many states
limiting recovery to the pecuniary losses of the decedents survivors caused by the decedents
death.

Minority Rule for wrongful death acts- measure damages by the loss to the decedents estate.
These allow for the estate to recover its lossessuch as lost income, funeral expenses, medical
expenses, or any other damages related to the decedents fatal injury.

Disposition- recovery allowed

Notes:

Typically all beneficiaries must adjudicate their claims in the same suit. The statute may
specifically provide how to proceeds recovered are to be divided among the beneficiaries.

Note p.355-56

Loses to the Decedents Estate

- Usually wrongful death recovery gives the survivors only the portion of the decedents
income that he or she would have spent on them- the financial support they lost by virtue
of the wrongful death

- Most survival statutes limit recovery to losses sustained before death, but that can put the
tortfeasor whose victim dies in better position than one whose victim is permanently
disabled. The victim who is permanently disabled is entitled to recover for lost earning
capacity for the remainder of his or her expected life. When the victim is killed, in most
states the tortfeasor has to pay only the portion of the earning that the decedent would
have contributed to survivors plus the decedents lost earnings from the tort until death

- The tortfeasor escapes liability for (a) earnings the decedent would have spent on himself
or herself and (b) earnings the decedent would have saved and thus passed on to his or
her estate.

- A few states avoid this situation by construing their survival statutes to permit the estate
to recover for the wealth the decedent would have likely accumulated during his or her
natural lifetime.

Loss of Consortium in Non-Fatal Cases


15
- Once courts began allowing for the loss of society in wrongful death cases, the question
naturally arose whether family members of seriously injured persons could recover such
damages in non-fatal cases.

- Most of the modern American cases confronting the issue have eliminated the
discrimination by recognizing a wifes cause of action for lost consortium.

- The majority of jurisdictions confine loss of consortium in non-fatal cases to spouses

- Some recent decisions have allowed childrens claims for loss of an injured parents
consortium and parents claims for loss of an injured childs consortium.

OTHER COMPENSATORY DAMAGE ISSUES


Interest

- Once a judgment is entered in a personal injury case, the amount normally is treated as a
debt owned by the defendant and interest begins to accrue at a rate that is usually set by
statute.

- Increasingly, courts have been inclined (or direct by legislatures) to award prejudgment
interest on the theory that the tortfeasors obligation to make the victim whole arises
when the injury occurs, or at least when the suit is filed.

Taxation

- Punitive damages are subject to federal income tax, but compensatory damages are not.

- In Rodriguez v. McDonald, the trial court forbade the parties from introducing evidence
relating to taxation. The court of appeals approved that procedure and said most
jurisdictions agree that jurors should be told nothing about taxation.

- Supreme Court has held that defendants in Federal Employers Liability Act (FELA) cases
are entitled to have the jury told that compensatory damages are not taxed.

Periodic Payments

- Problems with lump sum payments:

o Awards for pecuniary losses usually are reduced to present value on the theory that
the plaintiff can use the award to make more money by investing it. If the plaintiff
chooses to blow the money, the tort systems objective of making the injured party
self-sufficient is frustrated.

o It requires that all future losses be calculated at one moment in time, precluding
future adjustments for changes in earnings, medical costs, treatment methods,
inflation, and the plaintiffs condition

- A number of states permit or even require the court to arrange for periodic payment of
certain types of judgments (ex- medical malpractice or against governmental entities)

Structured Settlements

- Far more common are settlement agreements providing for periodic payments rather than
a lump sum.

16
- Parties often negotiate a structured settlement in which the defendant agrees to buy the
plaintiff an annuity that guarantees the plaintiff specified monthly or annual payments for
a specified period such as the rest of the plaintiffs life, a fixed number of years, or until
the plaintiff ultimately receives.

Cap on Damages

- In more than half of the states, legislatures have imposed caps on compensatory damages

- In some states the cap applies only to certain types of cases or certain types of damages

- Opponents of caps argue that they make the most seriously injured plaintiffs bear the
costs of tortious conduct while leaving whatever imperfections are thought to require
modification of the common law damages rules unaffected in the vast majority of cases
where damages do not reach the cap

- Proponents of caps say that they are a simple way of reducing the cost of tort liability and
insurance premiums

Alternative Compensation Schemes

- Best known alternative is workers comp. all states and the federal government have
statutes giving employers immunity from most tort liability for on-the-job injuries or
occupational diseases in exchange for their contributions to a fund of insurance plan which
pay injured worker benefits in amounts that are administratively determined.

- There are numerous exclusions, e.g., for intentional torts, and attempts by employees to
get out of the workers comp scheme and into the tort scheme generate numerous
litigation

- Another alternative is no fault insurance. The idea is that certain classes of injuries, such
as automobile accidents, are compensated by the victims own insurance, at least up to a
specified amount. The theory is that this is less costly than a system in which the
defendants insurance pays only after the victim establishes the defendants liability under
tort law.

- Occasionally Congress creates ad hoc compensation plans to bypass tort liability in


specific situations. I.e.- Victim Compensation Fund after 9/11

PUNITIVE DAMAGES

STATE FARM MUTUAL AUTO. INS. CO. v. CAMPBELL


Supreme Court of the United States, 2003

538 U.S. 408, 123 S.Ct. 1513

Procedural History- A jury determined that Campbell was 100% liable and returned a judgment
for $185,849. State Farm eventually paid the entire award, over an 8 year period, but the Utah
courts held that State Farm acted in bad faith in failing to promptly settle the suit against the
Campbells when they knew that Campbell was at fault. The Utah SC upheld an award of $1
17
million in compensatory damages for the Campbells emotional distress. It also upheld an award
of $145 million in punitive damages based on the findings that State Farms refusal to settle the
case against the Campbells was part of a nationwide scheme to meet corporate financial goals
by underpaying claims.

Facts- Campbell was involved in a collision with two other individuals. One was killed and the
other was permanently disabled. Campbells auto insurance company, State Farm, decided to
contest Campbells liability in the accident and declined settlement offers from the estates of the
others injured. Instead, they took the case to trial.

Issue(s)- Did the lower courts err in granting excessive punitive damages in violation of the due
process clause of the 14th Amendment?

Holding- Yes. The state of Utah did not have a legitimate concern in imposing punitive damages
to punish a D for unlawful act committed outside of its jurisdiction. The court erred in awarding
punitive damages to the Campbells to punish and deter conduct that was not related to the
Campbells harm. A D should be punished for the conduct that harmed the P, not for being an
unsavory individual or business. The punitive award of $145 million was neither reasonable nor
proportionate to the wrong committed, and it was an irrational and arbitrary deprivation of the
property of the defendant.

Rule(s)- The Due Process Clause of the Fourteenth Amendment prohibits the imposition of
grossly excessive or arbitrary punishments on a tortfeasor.

- 3 guideposts for courts to consider when reviewing punitive damages (BMW of N America
v. Gore):

o The degree of reprehensibility of the defendants misconduct

o The disparity between the actual or potential harm suffered by the plaintiff and the
punitive damages award

o The difference between the punitive damages awarded by the jury and the civil
penalties authorized or imposed in comparable cases.

Disposition- Reversed and remanded

Notes

-Punitive Damages may be awarded when the Ds conduct may be regarded as willful or wanton,
or where the Ds acts with malice or reckless disregard of the safety of others.

18
-Standard of Proof: Either by preponderance of the evidence OR clear and convincing (MS)
evidence

-Punitive damage award may be considered as part of the principal case or it may be
considered separately after any compensatory damage award is made.
-Many states now have damage caps on punitive damages. Some states require that a
significant portion of the punitive damages go to the state rather than the plaintiff.

- State Farm v. Mutual Auto. Ins. Co. v. Campbell

- Constitutional issue: Due Process Clause (14th Amendment)

- State Farm had a duty to settle because they had a chance to settle at the policy amount,
which was less than what the insured was exposed to. In this case, the exposure of
Campbell was much greater than the policy limit.

- BMW of North America:

o 3 guideposts

Degree of reprehensibility

Disparity between actual/potential harm to plaintiff and the punitive damage


award

The disparity between the punitive damage award and the civil penalties
authorized or imposed in comparable cases

- Factors in considering reprehensibility

o Type of harm- physical/economic mental state (indifference/reckless disregard)

o Vulnerability of the victim financially repeated or isolated misconduct

o Motive- malice, fraud, or accident

- Disparity between actual/potential damages and punitive award

o No bright-line ratio, but court will look unfavorably on awards that exceed a single
digit ratio.

o Larger the compensatory award, the smaller the ratio that is likely to be awarded

- Differences between civil penalties that are authorized or imposed in similar cases and the
amount of punitive awarded by the jury

- Other limits:

- State law cannot reach conduct of the defendant that is lawful in another state to punish
such conduct.
19
- Punitive damages must relate to relevant conduct

- Defendant must have notice that the conduct is subject to punitive damage claims

Scalia and Thomas dissented on the ground tha the const does not constrain the size of punitive
damages

20
CHAPTER 9- AFFIRMATIVE DEFENSES BASED ON
PLAINTIFFS CONDUCT
INTRODUCTION
- Affirmative defenses- any matter that, is pleaded and proved by the defendant, will defeat
or reduce the liability that plaintiff has otherwise established. The defendant has the
burden of proof to show that the plaintiff engaged in some kind of conduct that resulted
factually and legally in his/her own injury.

- In bearing the burden of proof, the defendant may rely on any relevant admissible
evidence introduced by any party to the action, not just evidence introduced by the
defendant.

- Examples of affirmative defenses:

o Release (as in a settlement agreement)

o Discharge in bankruptcy

o Res judicata- the thing has already been adjudicated. The court has already entered
a judgment and it cant brought again

o Estoppel- bared from bringing this claim due to some legal impediment

o Arbitration and award

o Fellow servant doctrine (Workers Comp)

o Duress, fraud, illegality, failure of consideration, accord and satisfaction, SOF,


waiver

o Laches

o Statute of Limitations

- Rule 8(c) of the FRCP requires a defendant relying on an affirmative defense to


affirmatively plead it, and lists as the affected defenses:

o Accord and satisfaction, arbitration and award, assumption of risk, contributory


negligence, discharge in bankruptcy, duress, estoppel, failure of consideration,
fraud, illegality, injury by fellow servant, laches, license, payment, release, res
judicata, SOF, SOL, waiver, and any other matter constituting an avoidance or
affirmative defense.

- In most states, the defendant must plead and prove contributory or comparative
negligence and assumption of risk

- Traditional rule was contributory negligence- negligence of the plaintiff in being a factual
and legal cause of his or her own injury is a complete bar to plaintiffs recovery (recovered
nothing)

- Courts attempted to mitigate harshness: kind of an all or nothing proposition

21
o Ignoring plaintiffs negligence or applying a double standard

What was negligent for the D, the same conduct was not deemed to be
negligent for the P. The court would say that the duty you owe to yourself is
not as important as the duty that you owe to others. (Not very sensible)

o Last clear chance doctrine

If the P is negligent in putting herself in a position of peril, and the D is


negligent in failing to perceive that peril and to avoid it, then the D has the
last clear chance to avoid the accident, then the Ps negligence is ignored
and all of the blame falls on the D.

o Plaintiffs negligence is slight in comparison to defendants

o Defendant was grossly negligent or willful and wanton (reckless)

o Passive v. active negligence

If the P is passively negligent and D is actively negligent, then D should be


responsible for the loss

- All or nothing approach:

o Either P or D bore the entire loss, even though both were at fault

- Under common law rules, if the tort is intentional, the Ps negligence does not matter

- If there is an intentional tort, the damages might be able to be mitigated (inducement or


provocation)

- Comparative Negligence

o Liability is apportioned or shared according to assessment of fault or responsibility

o Mississippi was the first state to adopt the comparative negligence standard (1910)-
it was adopted by statute

o Not until the 1960s did other states begin moving toward a generally applicable
comparative negligence regime for torts

o By court adoption or by legislation

o Mississippi did so by legislation.

o 5 contributory jurisdictions:

Alabama

Maryland

North Carolina

Virginia

22
Washington, D.C.

CONTRIBUTORY AND COMPARATIVE NEGLIGENCE

HOFFMAN v. JONES
Supreme Court of Florida, 1973

280 So.2d 431

Procedural History- The trial judge denied Ps request to instruct the jury on comparative
negligence. The trial judge gave the standard contributory negligence instruction, and the jury
returned a verdict for the defendant. P appealed to the intermediate appellate court, which
certified the following question the state supreme court: Whether or not the Court should replace
the contributory negligence rule with the principles of comparative negligence?

Facts- Ps husband was killed in a traffic accident that resulted from a combination of his own
negligence of the other driver.

Issue(s)- Did the trial court err in denying Ps request to instruct the jury on comparative
negligence?

Holding- Yes. A plaintiff in an action for negligence will no longer be denied any recovery
because of his contributory negligence. If it appears from the evidence that both P and D were
guilty of negligence which was, in some degree, a legal cause of the injury to the P, this does not
defeat the Ps recovery entirely. The jury in assessing damages as in the jurys judgment the
negligence of the D caused to the P. In other words, the jury should apportion the negligence of
the P and the negligence of the D; then in reaching the amount due the P, the jury should give
the P only such amount proportioned with his negligence and the negligence of the D.

Rule(s)-

1. To allow a jury to apportion fault as it sees fit between negligent parties whose negligence
was part of the legal and proximate cause of any loss or injury, and

2. To apportion the total damages resulting from the loss or injury according to the
proportionate fault of each party

Analysis-

Disposition- Remanded to trial court for a decision consistent with this ruling

23
Notes-

- Should Florida judicially adopt comparative negligence regime in place of contributory


negligence rule?

o Yes the court did adopt this as the new rule

- Contributory negligence is a judicially created rule

- Generally regarded as unjust and harsh

- Movement has been away from contributory negligence toward comparative negligence

- Justifications for contributory negligence are not valid in the modern era

- Comparative negligence is more equitable.

- Contributory negligence rule is often disregarded by juries.

- Efforts by the courts to ameliorate the harshness of the rule are unsatisfactory

- Florida adopted pure comparative fault

o Plaintiff may recover damages in the amount of the percentage of responsibility of


the defendant for the accident after apportioning responsibility between the plaintiff
and the defendant (if the plaintiff is also at fault). It doesnt matter what the % are,
as long as they add up to 100%.

- Modified 51% system, and modified 50% system

o If the Ps negligence exceeds a certain %, then the D is not required to pay for any
damages.

o One system says if there is a 50/50- P gets nothing- Must be 50% or less for P to
recover

o The other says if the P is above 51%, P gets nothing If P is equally or more
negligent than D, then P does not recover

o Majority of states that use modified rules, use the 50% standard.

Look at n.7 p.385, regarding set-off

Jury instructions in modified systems- do you tell the jury about the effect of ps negligence?
tendency to tell the jury, but maybe half and half

24
WASSELL v. ADAMS
United States Court of Appeals, 7th Circuit, 1989.

855 F.2D 849

Page 386

Procedural History- Trial Court jury awarded P $850,000 worth of damages, which was reduced to
$25,000 since she was ruled to be 97% at fault. Ps attorney moved for JNOV on the grounds
that she had either not been negligent as a matter of law or that her negligence was immaterial
because the Ds negligence was willful and wanton in their disregard for her safety. Judge
denied to motions and P appeals.

Facts- P was a tenant of the Rin-Ric Motel, which was owned by the defendants. This was located
in a bad area of town. P was awoken by a knock at her door around 1:00 a.m. A well dressed
black man was on the other side of the door. He asked for another woman, and was told that
there was nobody by that name there. He then asked for a glass of water to which the P obliged.
When she returned from fetching the water, the man was in her room. He proceeded to rape
her. Subsequently, P has suffered great emotional distress.

Note- After the date the action was filed, IL adopted a new statute that bars a P from recovery
when they are more than 50% negligent in causing the injury for which recovery is sought.

Issue(s)- Did the district court judge err in denying Ps request for JNOV? Did the district court
judge err in not allowing Ps motion for a new trial?

Holding- No. No rational jury could find that the Ds consciously disregarded a high probability of
serious physical harm. Thus, their negligence was not willful or wanton, and must be considered
simple negligence.

Rule(s)- The common law refused to compare the Ps and the Ds negligence. The negligent P
could recover nothing, unless the Ds culpability was of a higher degree than simple negligence
(willful or wanton).

-Appellate Courts are not the trier of fact, and they are only authorized to upset the jurys
apportionment if persuaded that the trial judge abused his discretion in determining that the
Jurys verdict was not against the clear weight of the evidence.

Reasoning/Analysis- It is unlikely that a warning would have averted the attack. P testified that
she thought the man who had knocked on the door was her fianc. Thinking this, she would
have opened the door no matter how dangerous she believed the neighborhood to be.

25
Disposition- Affirmed.

Notes-

- Appeal from denial of motions for JNOV

- What happens to the distinctions at common law involving willful and wanton and
gross negligence?

- Do these survive insofar as preventing plaintiffs negligence from being considered in


reduction of her recovery?

- What should the effect of conscious disregard of anothers safety be in a comparative


negligence system?

- How do juries apportion damages between a P and D when both have acted negligently?

- Is it really a matter of comparing costs of avoidance?

- Is it a matter of declining the degree of departure from the norm? Is it mostly an intuitive
matter?

- Ameliorative doctrines- losing favor

- Intentional torts

o Comparative fault, good or bad- does it depend on the facts?

E.g.- mutually agree fight (or in the circumstance of fighting words


exchanged) v. Wassell rapist situation?

FAILURE TO AVOID CONSEQUENCES; FAILURE TO MITIGATE


DAMAGES

DARE v. SOBULE
Supreme Court of Colorado, 1984

674 P.2d 960

Procedural History- Petitioners (Dare) brought this action against Sobule to recover damages for
the wrongful death of their son. At trial, two witnesses testified that decedent was not wearing a
protective helmet at the time of the accident. Petitioners did not object to the introduction of the
evidence. The trial court refused to enter the evidence to the jury that failure to wear a helmet
while operating a motorcycle is not contributory negligence. The jury found that both the
decedent and the defendant were negligent. The decedent was found to be 80% liable and the
defendant was 20% liable. Because Colorado is a modified comparative fault state, the plaintiff
took nothing and appealed. The Circuit court of appeals affirmed.

26
Facts- Tracy Dare, decedent, was riding a motorcycle when Sobule, who was driving an
automobile made a left turn in front of Dares motorcycle. The motorcycle struck the vehicle and
Dare was thrown over the car, landed on his head, and died as a result of head injuries.

Issue(s)-

1. Did the trial court err in refusing to instruct the jury that failure to wear a helmet when
riding a motorcycle was not contributory negligence?

2. Whether the court should impose a standard of conduct for those riding motorcycles?

Holding-

1. Yes. Under the law of comparative negligence in the state of Colorado, evidence of a
plaintiffs failure to wear a protective helmet while riding a motorcycle is inadmissible to
show negligence on the part of the plaintiff or to mitigate damages.

2. No, for three reasons. First, a D should not diminish the consequences of his negligence by
the failure of the injured party to anticipate Ds negligence in causing the accident itself.
Second, a defense premised on an injured partys failure to wear a protective helmet
would result in a windfall to tortfeasors who pay only partially for the harm their
negligence caused. Third, allowing the defense would lead to a battle of experts as to what
injuries would have or would not have been avoided had the P been wearing a helmet.

Rule(s)- The General Assembly of Colorado has not mandated the use of protective helmets as a
standard of conduct.

- Under the law of comparative negligence in the state of Colorado, evidence of a plaintiffs
failure to wear a protective helmet while riding a motorcycle is inadmissible to show
negligence on the part of the plaintiff or to mitigate damages.

Analysis- The improper admission of evidence of failure to wear a protective helmet provides
rational explanation for the jurys findings that the decedent was 80% liable.

Disposition- Reversed and remanded

Notes-

- Evidence of failure to wear motorcycle helmet or seatbelt is not admissible to establish


contributory negligence or to reduce the plaintiffs damages. You dont need to anticipate
the negligence of others.

- -Rational behind Dare:

1. Rule avoids windfall to D in allowing him to avoid full consequences of his


misconduct

27
2. Avoids battle of the experts (least compelling of the three made here)

3. Defendant should not diminish consequences of his negligence by failure of


innocent party to anticipate the defendants wrongful conduct

- Other arguments

o Absence of legislative mandate

o No duty to mitigate before injury occurs

HUTCHINS v. SCHWARTZ
Supreme Court of Alaska, 1986

724 P.2d 1194

Procedural History- P sued D for $275K in compensatory damages. He filed a motion in limine to
exclude evidence of his failure to wear a seatbelt. This request was denied by the trial judge. The
judge granted Hutchins motion for a directed verdict on the seatbelt issue. The jury was
instructed to disregard all evidence relating to Hutchins failure to wear a seatbelt. The jury
found that Hutchins was 40% liable and Schwartz was 60% liable. The awarded Hutchins
$1,937.09 in damages. Hutchins moved for JNOV and/or a new trial. This motion was denied and
Ps appealed.

Facts- Hutchins was injured in an automobile accident with Schwartz. Hutchins was not wearing
his seatbelt at the time of the accident and sustained cuts on his head, bruises on his chest,
knee and wrist, and a broken big toe.

Issue(s)-

1. Did the trial court err by admitting evidence of Hutchins failure to wear a seatbelt?

2. Did the trial court err by denying his motion for JNOV and/or new trial?

3. Whether the court should impose a duty upon a person to wear a seatbelt when driving a
car equipped with one?

Holding-

1. No. If under the facts and circumstances of the case a reasonably prudent person would
have used a seatbelt and if plaintiff suffered more severe injuries as a result of not
wearing a seatbelt, then the jury should be permitted to consider the factor in assessing
damages.
28
2. No. After reviewing the evidence, reasonable minds could differ as to whether Hutchins
headlights were on or off. Additionally, the jury could have found that Hutchins was
traveling too fast for the road conditions. In conclusion, there is an evidentiary basis for
the jurys finding that Hutchins was 40% comparatively negligent.

3. Yes. Automobile accidents are foreseeable. The fact that many motorists do not wear
seatbelts may suggest that a failure to wear a seatbelt does not violate a substantial
standard of care. However, the fact that a majority of people act in a certain manner does
not make that conduct reasonable, especially when that conduct involves an unnecessary
risk.

Rule(s)-

- Foley v. City of West Allis- In light of the realities of the frequency of automobile accident
and the extensive injuries they cause, the general availability of seatbelts, and the public
knowledge that riders and drivers should buckle up for safety, those who fail to use
available seatbelts should be held responsible for the incremental harm caused by their
failure to wear available seatbelts. A plaintiff only recovers damages for injuries caused by
defendant and not for those that plaintiff could have prevented by wearing a seatbelt.

- Insurance Co. of North America v. Pasakarnis-In light of the importance of the seatbelt as a
safety precaution and the minimal effort required to fasten an available seatbelt, the court
concluded that failure to wear one could be a pertinent factor for the jury to consider in
determining damages.

Analysis-

Disposition- Affirmed

Notes-

- This is a minority view. Many states forbid introduction of seatbelt evidence to show
comparative fault negligence. In some jurisdictions it can be introduced in products
liability, and a few other areas.

- Failure to avoid consequences is generally not required. You are not generally required to
anticipate the negligent conduct of another person. Actions that could have been taken by
P beforehand. Generally they do not have a negative impact on Ps recovery.

- Failure to mitigate damages refers to what you do after the injury happens. Plaintiff has a
duty to act reasonably after the damage is done in order to recover. If he fails to do so, the
D may be entitled to reduction of damages. He is not liable for the value of the enhanced
injury that is attributable to the Ps failure to mitigate. Use comparative fault to allocate
that value.

- Comparative (or contributory) negligence: Pre-accident conduct by the victim that was a
cause of the accident and hence all of the injuries or damages. Just because you are in a
comparative negligence system, does not mean that you do not talk about the Ps
contributory negligence in the accident.

- Notes 2 and 3 on page 401

29
ASSUMPTION OF THE RISK

BENNETT v. HIDDEN VALLEY GOLF AND SKI, INC.


United States Court of Appeals, Eighth Circuit, 2003

318 F.3d 868

Procedural History- Bennetts action against Hidden Valley alleged that Hidden Valley was
negligent in the design, maintenance, and staffing of its skiing facilities; in the supervision of its
customers so as to prevent or cure dangers created by such business invitees; in providing its
customers with unrestricted access to advanced and intermediate ski areas without assessing
(their) ski aptitude, ability, or experience; in permitting obstructions, including tress and jumps,
to exist in the path of skiers at a time when (it)should have known that such obstructions
posed a hazard or risk of injury; in failing to warn of dangers and obstructions which it knew or
reasonably should have known were present at its facilities and ski slopes; and in failing to
guard against, barricade, protect, or cushion knowable obstructions in the path of skiers upon its
ski slopes. Hidden Valley denied negligence and raised assumption of the risk as a defense. The
case proceeded to a jury trial. At the close of all of the evidence, Bennett moved for a judgment
as a matter of law, claiming that Hidden Valley had not established its affirmative defense as
assumption of the risk. The DC denied the motion. The jury returned a verdict in favor of Hidden
Valley. Bennett appealed.

Facts- Bennett went with two older male friends to Hidden Valley for a midnight ski lesson. She
was 16 years old at the time. She had limited experience as a skier. While she was skiing down a
slope marked for intermediate difficulty, she fell on a bump in the ski slope. She was thrown
about 5 feet forward and hit the ground limp. The bump on the slope was not intentionally
created by Hidden Valley, but had formed as skiers and snow boarders cut across the slope and
moved the snow. Bennett claims injuries as a result of the accident, including brain damage and
a diminished future earning capacity.

Issue(s)-

1. Did the DC err in instructing the jury to find for Hidden Valley if the conditions Bennett
encountered were risks inherent in the sport of skiing?

2. Did the DC err in denying Bennetts motion for judgment as a matter of law because
Hidden Valley did not make out an assumption of risk defense?

Holding- No and no. Under Missouri law, a voluntary skier assumes the risk inherent in or
incidental to skiing, regardless of her subjective knowledge of those risks. The proprietor of a ski
area has no duty to protect a skier from those risks incidental in skiing. By directing the jury to
find for Hidden Valley if it determined that the conditions on the ski slope at the time Bennett

30
was injured were inherent risks of skiing, Instruction 7 fairly and adequately submitted the issue
to the jury. The DC therefore did not abuse its discretion by giving this charge.

Rule(s)- Affirmative defense in the State of Missouri:

- The defense applies where the parties have voluntarily entered a relationship in which the
plaintiff assumes well-known incidental risks.

- A plaintiffs consent to assume the risk is implied from the act of electing to participate in
the activity and as to those risks, the defendant has no duty to protect the plaintiff

1. A participant in sport accepts those hazards that reasonably inherent in the sport so far as
they are obvious and usually incident to the game

2. The patron subjects himself to the dangers necessarily and usually incident to and
inherent in the game. This does not mean that he assumes the risk of being injured by the
proprietors negligence but that by voluntarily entering into the sport as a spectator but
that by voluntarily entering into the sport as a spectator he knowingly accepts the
reasonable risks and hazards inherent in and incident to the game.

Analysis-

Disposition- Affirmed

Notes-

- Assumption of the risk

- Three uses of the term:

o Express assumption of the risk

When there is a written assumption of the risk in writing by the P, who waives
the negligence claim prior to the injury (ex- amateur drag racing)

o Implied primary assumption of the risk

Come about in the context of comparative negligence

In the nature of a lack of duty- more like a no duty rule

Used to be a complete bar to recovery

o Secondary implied assumption of the risk

Most often associated with assumption of the risk

A knowing, involuntary and unreasonable knowing of the risk

Use to be a complete bar to recovery

31
- Express assumption of the risk

o Contractual in nature- an agreement not to hold someone liable for negligence prior
to some activity or occurrence. No duty issue.

- Implied Primary Assumption of the Risk

o No duty

o Plaintiff assumes any risk inherent in an activity (typically as a participant or


spectator in a sport)

o Objective test- is the risk inherently dangerous? And is it generally known?

- Secondary Implied Assumption of the Risk

o Plaintiff has knowledge and appreciation of the danger posed by defendants


negligence and voluntarily chose to expose himself to the danger (most states that
use comparative fault have abolished this kind, still applies in AL), subjective and
arguably stronger than arguing contributory negligence

- Mississippi has abolished assumption of the risk as a separate defense that acts as a total
bar to recovery and treats the plaintiffs conduct under general principles of comparative
negligence, except in products liability cases

o In products liability cases in MS, assumption of the risk is a complete bar to


recovery

- Note 4

IMPUTED CONTRIBUTORY FAULT

CONTINENTAL AUTO LEASE CORP. v. CAMPBELL


Court of Appeals of New York, 1967.

19 N.Y.2d 350

Page 411

Procedural History- Trial Court jury awarded judgment for Continental, even though both drivers
were found to be contributory negligent. Appellate Division affirmed.

Facts- Continental Auto Lease Corp. rented a car to a gentlemen by the name of Kamman for a 4
day period. During said rental, Kamman was involved in an automobile accident with Ralph B.
Shepard. Upon the trial, the jury found both drivers negligent, but returned a verdict for
Continental. Shepard has since died, and Campbell is his administrator.

Issue(s)- Did the trial court err in ruling that the negligence of Kamman was not imputable to
Continental? Or if it was imputable to Continental, should Continentals recovery have been
barred by contributory negligence.
32
Holding- No. Continental had no interest in where or when the vehicle was driven and no
relationship to Kamman consistent with the inference that it had the right to control in any
manner Kammans conduct as a driver. Accordingly, Kammans negligence should not be
imputed to Continental to bar its recovery in this action.

Rule(s)-1. One who is injured by the negligent operation of a motor vehicle should have
recourse to a financially responsible defendant.

2. If a car owners relationship to the driver of his car is such that a degree of physical
control over the driver can reasonably be deemed to exist, then the negligence of the
driver can be imputed to bar the owners recovery against a negligent 3rd party.

Reasoning/Analysis-

Disposition- Affirmed.

Notes- Imputed contributory negligence

- Similar in concept to vicarious liability

- But purpose is different- to restrict liability rather than broaden liability

- Imputed contributory negligence is when the plaintiff is held accountable in terms of


assessing responsibility for an injury for the negligence of someone else. That persons
negligence is treated as if it were the plaintiffs own negligence in terms of its effect on
the plaintiffs recovery against a third person. If P were D, could he be held vicariously
liable? If yes, then his claim can be imputed

- If the employer is vicariously liable, when the employer is the P, the employees negligence
can be imputed to the employer to offset the damages of a third party

- Most common ways to impute damages (Both Ways Rule)

o Employer/ Employee

o Employer/ Independent Contractor

o Joint Venture or Enterprise (Partnership in business)

- Imputed damages

o Derivative claims- When the plaintiffs cause of action is derivative- based on an


injury to someone else (consortium, parents claim for medical expenses incurred
for child (not always), wrongful death and survival, bystander)

- Case Notes

o NY 1967:

Is the fault of a bailee of a commercial bailor imputable to the bailor in the


bailors claim against the bailees co-tortfeasor?

o No imputed negligence to commercial bailor for bailees negligence in claim by


bailor against third person for negligent damage to bailors property.

33
WHITE v. LUNDER
Supreme Court of Wisconsin, 1975.

66 Wis.2d 563

Page 414

Procedural History- Trial Court ruled in favor of defendant, ruling that P/Appellant, Lloyd White,
was precluded from recovery because he and his wife were 63% contributory negligent.

Facts- Rosemary White and her husband, Lloyd White, as well as James Lunder were involved in
a boating accident. The trial court jury apportioned the causal negligence among the parties to
Rosemary White, 30%; Lloyd White, 33%; and Lunder, 37%. The Wisconsin statute for
comparative negligence provides that contributory negligence shall not bar recovery if such
negligence was not as great as the negligence of the person against whom recovery is sought,
but any damages allowed shall be diminished in the proportion to the amount of negligence
attributable to the person recovering. The trial court judge combined the negligence
apportionment of Plaintiff and his wife (63% total). Accordingly, he was barred from recovery.

Issue(s)- Did the trial court judge err in combining the negligence of both the Plaintiff and his
wife for the purposes of comparing negligence with that of the third party for a claim for medical
expenses and loss of consortium? Are these claims derivative?

Holding- Yes. These were derivative actions. A workable construction, consistent with the
statute, that will allow recovery in derivative actions where the causal negligence of the person
against whom recovery is sought is greater than either the husband or wife can be accomplished
by reducing the entire award both medical expenses and loss of consortium by the percentage of
negligence attributed to the injured spouse; and further reducing the entire award by the
percentage of causal negligence attributable to the claiming spouse. By this method the person
who was found to be causally negligent in greatest degree cannot escape all liability but his
liability is decreased by an amount proportionate to the two other tortfeasors.

Rule(s)- A person should not be precluded for summary judgment, so long as his contribution to
the negligence is less than that of the person to which he seeks to recover, when dealing with
derivative claims. Additionally, when dealing with multiple parties who are contributory
negligent to an accident, the apportionments should not be added together for the purposes of
deciding whether recovery should be precluded.

Reasoning/Analysis-

Disposition- Reversed. Award for Plaintiff/Appellant consistent with this opinion.

Notes-

34
- What is the effect of the wifes negligence on husbands consortium claim against
defendant?

- Wisconsins early comparative neg standard- if Ps negligence is equal to or > Ds


negligence= no recovery

- Note 2 after case

- Imputed negligence doesnt apply to claims by persons against others as to whom


negligence might be imputed when an outsider makes the claim

35
CHAPTER 10- OTHER AFFIRMATIVE DEFENSES
STATUTES OF LIMITATIONS

JOLLY V. ELY LILLY & CO.


Supreme Court of California, 1988.

Page 418

Procedural History-

Facts- P was born in 1951 from a mother who ingested DES for the prevention of miscarriage. In
1972, P learned that DES daughters could suffer injuries from DES in utero. She went for a check
up and was diagnosed as having adenosis. In 1976, she had an abnormal pap smear and
underwent a procedure to remove the abnormal tissue. In 1978, P underwent a complete
hysterectomy and a partial vaginectomy in order to remove malignancy. As of 1972, P was
aware or at least suspected, that her condition was a result of her mothers ingestion of DES
during pregnancy. Starting in 1972, P attempted to find the manufacturer of the drug.
Unfortunately, the doctor who had provided the medication had died, and the pharmacist who
filled the prescription did not have records indicating the brand of DES prescribed. There is a 1
year statute of limitations for an action for injury caused by the wrongful act or neglect of
another.

Issue(s)- Did the trial court err in ruling that the Plaintiffs claim was barred by the statute of
limitations?

Holding- No. The limitations period begins when the P suspects, or should suspect, that she has
been wronged. Here, P suspected as much no later than 1978. Because she did not file suit until
1981, he suit, unless otherwise saved, is time-barred.

Rule(s)- Discovery Rule- Provides that the accrual date of a cause of action is delayed until the
P is aware of her injury and its negligent cause. A plaintiff is held to her actual knowledge as well
as knowledge that could reasonably be discoverable through investigation of sources open to
her.

Once the P has a suspicion of wrongdoing, and therefore an incentive to sue, she must decide
whether to file suit or sit on her rights. So long as a suspicion exists, it is clear that the P must
go find the facts; she cannot wait for the facts to find her.

Doe Party Rule- Pursuant to Cal. Code of Civ Pro. Section 474, P could have filed suit against a
Doe Party. In these actions, from the time such a complaint is filed, the P has 3 years to
identify and serve the defendant.

Reasoning/Analysis-

Disposition- Affirmed.

36
Notes-

- Statutes that prescribe the period of time in which an action must be brought or which will
then be time-barred

- Affirmative defense. Defendant must plead and prove the facts that would establish the
existence of a time bar.

- Awareness or knowledge is both actual and constructive

- Constructive knowledge is knowledge that is reasonably discoverable through


investigation of sources available to plaintiff

- How most states start the SOL:

o Last point in time of continuing tort

o Time of injury

- Jolly v. Eli

o Discovery rule- accrual date of a cause of action is delayed until the plaintiff is
aware of her injury and its negligent cause

o SOL in this case was when she has enough facts that would give her reasonable
suspicion there was a causal connection between her injury and someone elses
misconduct or negligence. She had been suspicious since at least 1972 of the DES
product. Her SOL had at least run out by 1972.

o SOL avoids stale claims

o Ignorance of legal significance of facts- statute continues to run

o SOL begins to run when the plaintiff has a suspicion or should have a suspicion that
someone has injured her by wrongdoing

o Note 4- 419 (footnote in case)

Ds argue that the statute should commence when the P knows of her injury
and its factual cause.

o Once P has knowledge sufficient to excite suspicion of inquiry, the statute begins to
run. The P must take steps necessary to preserve her right to a remedy by
investigating and bringing suit within the statutory period. (Discovery Rule) -----More
commonly applied version

o Policy reasons for SOL:

Protect D from defending stale claims

Loss of evidence

Inability to plan for future contingencies

Plaintiffs should diligently pursue their claims.

37
o Effect of change in the law after the statute has otherwise run?

None. The claim is time-barred. A change in the law does not revive a claim
that is time barred.

o Policy reasons for not reopening time-barred cases

Case management problems for court

Need for finality for parties, particularly defendants

Stale claims

o Doe Party Complaints

o Discovery Rule

Not applied in every state and limited to specific kinds of cases in other
states

Often SOLs do not run against minors or others with mental incapacity.

o Note 2

When the tort occurs, look at the statutes

o Note 3

Facts establishing wrongdoing

Or facts establishing cause of injury whether or not wrongdoing is


suspected or known

o Ps may have other theories available to them.

o Ps are free to seek change in the common law when they believe the law to be
unjust or inadequate.

FELTMEIER v. FELTMEIER
Supreme Court of Illinois, 2003

798 N.E.2d 75

Procedural History- Trial court denied Roberts motion but certified all three issues for
interlocutory appeal. The appellate court affirmed and Robert appealed. On the first issue, the SC
held that the conduct alleged was extreme and a there was a valid claim of IIED.

Facts- Plaintiff, Lynn Feltmeier ,and Defendant, Robert Feltmeier, were married in 1986 and
divorced in 1997. In 199, Lynn sued Robert for IIED. She alleged that he engaged in a pattern of
emotion and physical abuse which began shortly after the marriage and continued through and
after the divorce. Lynn alleged that Robert hit her, kicked her, prevented her from leaving the
house, threw things at her, abused her verbally, and, after the divorce, stalked her. Robert
38
moved to dismiss on the ground that the complaint failed to state a claim and that the claim was
barred by the SOL.

Issue(s)- Did the appellate court err in holding that the conduct was a continuous tort and the
SOL of limitation did not being to toll until after the divorce was final?

Holding- No. The two year SOL for this action began to run in August of 1999, because Lynns
complaint includes allegations of tortious behavior by Robert occurring as late as that month.
Applying the continuing tort rule to the instant case, Lynns complaint, filed August 25, 1999,
was clearly timely and her claims based on conduct prior to August 25, 1997, are not barred by
the applicable statute of limitations.

Rule(s)-

- Continuing tort rule- where a tort involves a continuing or repeated injury, the
limitations period does not begin to run until the date of the last injury or the date the
tortious acts cease

- General SOL rule- where there is a single overt act from which subsequent damages
may flow, the statute begins to run on the date the defendant invaded the plaintiffs
interest and inflicted injury, and this is so despite the continuing nature of the injury

- Pavlik v. Kornhaber holding- Illinois courts have said that in many contexts, including
employment, repetition of the behavior may be a critical factor in raising offensive acts to
actionably outrageous ones. It may be the pattern, course and accumulation of acts that
make the conduct sufficiently extreme to be actionable, whereas one instance of such
behavior might not be. It would be logically inconsistent to say that each act must be
independently actionable while at the same time asserting that often it is the cumulative
nature of the acts that give rise to IIED. Likewise, we cannot say that cumulative
continuous acts may be required to constitute the tort but that prescription runs from the
date of the first act. Because it is impossible to pinpoint the specific moment when enough
conduct has occurred to become actionable, the termination of the conduct provides the
most sensible place to being the running of the prescriptive period.

Analysis-

Disposition- Affirmed

Notes-

Continuing tort:

o Effect on running of SOL?

o When the conduct of a continuing nature stops, thats when the SOL starts running

o Problem- what conduct constitutes a continuous tort?


39
- Continuing tort- a tort involving continuing ot repeated injury. Generally involves a pattern
of conduct as opposed to one act

- SOL does not begin to run until the date of the last injury or the date the tortious acts
cease.

- Distinguishing continuing ill effects from continuing unlawful conduct

- Single acts v. repeated conduct that is seen as a continuous whole

- Pattern of conduct that as a whole can be seen as extreme and outrageous conduct
necessary to establish IIED.

- - Med Mal or Legal Mal- Note 2

STATUTES OF REPOSE

BRADWAY v. AMERICAN NATIONAL RED CROSS


United States Court of Appeals, Eleventh Circuit, 1993

992 F.2d 298

Procedural History- Plaintiff and her husband filed a complaint alleging that P contracted AIDS
during her 1983 blood transfusion. Ps sought compensatory damages, contending that the Red
Cross was negligent in screening blood donors and in testing blood samples for the presence of
HIV. The Red Cross moved the DC to dismiss the action as barred by Georgias SOL and ultimate
repose for med mal suits. The DC concluded that under GA law, an action against a blood bank
for the negligent collection and supply of human blood is an action for med mal, and dismissed
the case as barred by GAs statute of repose. Ps argue that the action did not accrue until the
wrong was completed, i.e., when P became infected, and that it is a jury question as to when P
became infected with the AIDS virus.

Facts- Plaintiff underwent reconstructive surgery for facial birth defects. She received 2 units of
whole blood by transfusion after the surgery. The hospital obtained the blood from an American
National Red Cross blood bank. The Red Cross had no direct connection with the Plaintiff.

Issue(s)- Did the DC err in classifying the case as a medical malpractice suit barred by GAs
statute of repose rather than classifying it as an ordinary negligence case?

Holding- No.

Rule(s)-

40
- O.C.G.A. 9-3-70- The term action for medical malpractice, means any claim for
damages resulting from the death of injury to any person arising out of:

o Health, medical, dental, or surgical service, diagnosis, prescription, treatment, or


care rendered by a person authorized by law to perform such service or by any
person acting under the supervision and control for the lawfully authorized person;
or

o Care or service rendered by any public or private hospital, nursing home, clinic,
hospital authority, facility, or institution, or by any officer, agent, or employee
thereof acting within the scope of his employment

- O.C.G.A. 9-3-71-

o Except as otherwise provided in this article, an action for medical malpractice shall
be brought within two years after the date on which an injury or death arising from
a negligent or wrongful act or omission occurred.

o Subject to the discovery rule, in no event may an action for medical malpractice be
brought more than 5 years after the date on which the negligent or wrongful act or
omission occurred.

- SOL- governs the time within which legal proceedings must be commenced after the cause
of action accrues.

- Statutes of Repose- limits the time within which an action may be brought and is not
related to the accrual of any cause of action. The injuries need not have occurred, much
less have been discovered.

Analysis- The plain language of the repose statute indicated that the period begins on the date
on which the negligent or wrongful act or omission occurred.

Disposition- Affirmed

Notes-

- Statutes of Repose- limits time within which action must be brought without regard to
when the cause of action accrued or whether it has yet accrued.

- Different from an SOL-

o SOL

o SOR- cause of action might have no accrued- runs from the act, which might be
wrongful which might not have triggered any results yet.

- Thus, lack of knowledge of the existence of the cause of action does not toll a statute of
repose.

- In this case, the wrongful act was the blood transfusion the SOR was triggered at that
date, although she did not test positive for HIV/AIDS for another 5 years.

41
- Filing a case of action against a blood bank is a med mal case, and is barred by 2 years. In
addition, the SOR bars the claim because it gives 5 years from the date of the act.

- Examples of types of cases in which legislatures have enacted statutes of repose:

o Medical malpractice

o Architects and building contractors

o Products liability

- Negligence involved in the collection and supplying of blood gives rise to a med mal action

- SOR began running on the date that the negligent act or omission occurred, not when the
negligence was discovered by plaintiff or when injury occurred.

- GA statute bars claims for med mal unless brought within 2 years of the negligent or
wrongful act or omission. (ordinary SOL)

- But in no event may such a claim be brought in more than 5 years after the date on which
the negligent or wrongful act or omission occurred

- Note 3-

EXPRESS ASSUMPTION OF RISK

HOLZER v. DAKOTA SPEEDWAY, INC.


Supreme Court of South Dakota, 2000

610 N.W.2d 787

Procedural History- This case arose from a personal injury accident, when a race cars wheel
detached and hit and injured the plaintiff. The circuit court granted Ds motion for summary
judgment based on a pre-accident released signed by P.

Facts- Before entering the pit area of the racetrack (to serve on the pit crew for one of the
racers), Holzer was requested by Speedway to sign as Release and Waiver of Liability,
Assumption of Risk and Indemnity Agreement. All individuals wishing to enter the pit area were
required to sign the release form. This document provided that the signees covenant not to sue
the track owners, insurers and discharge them from any and all liability. Signing the release was
a condition to being allowed into any restricted area such as the pit, and applied to anyone
competing, officiating, observing, working for, or participating in races at the speedway. At the
bottom of the release, printed by each signature line was the heading I HAVE READ THIS
RELEASE. P signed and submitted it. He had done this twice before as well.

Issue(s)- Did the lower court err in granting Ds motion for summary judgment based on a pre-
accident release signed by the plaintiff?

42
Holding- No. The release in this matter does not involve a matter of public policy, but rather a
private agreement between individuals. P participated in Speedway races as a form of
recreation- he was a volunteer and was therefore not compelled in any way to enter the
Speedway racing pit area. In addition, there was no evidence in the record that P was denied the
opportunity to step out of line at any time and read the release. He was not forced to sign it. If he
had problems reading/understanding the release (as his father indicated), he should have asked
someone to read and or explain it to him. It is his own fault that he didnt read it READ SHIT
BEFORE YOU SIGN IT!!!

Rule(s)-

- To be valid, a release must be fairly and knowingly made

- One who accepts a contract is conclusively presumed to know its contents and to assent
to them, in the absence of fraud, misrepresentation, or other wrongful act by another
contracting party

Analysis-

Disposition- Affirmed

Notes-

- Express Assumption of the Risk Exculpatory Clauses

- Explicit, Clear, conspicuous (understandable language)

- May not disclaim willful negligence or intentional wrongdoing

- May not disclaim liability when a matter of public interest is involved

- May not disclaim liability for utilities or quasi-public entities supplying essential services

- Meaningful opportunity to be apprised of the waiver

- Fairly and voluntarily made

- But signatures are not mere ornaments to contracts

- See note 1

o Some states choose to limit the ability of parties to waive personal injury claims in
some situations

- Commercial parties

o Relative bargaining strength

- Consumers

o Not equal bargaining power- adhesionary situation

43
- UCC warranty provisions

o Opportunities were parties may waive warranty options concerning negligence and
or product liability

- Notice of claims

o Sometimes, before filing suit against a state or local govt notice must be provided
within a specified period following the injury as a condition of the right to sue.

- Types of Comparative Negligence Regimes:

o Pure

Ms

LA

FL

Neglience od P is never a bar to recovery

o Modified

50%

P may recover as long as the Ps negligence is less than that of the D

P must be less than 50% negligent

TN

GA

AR

51%

P may recover only if the Ps negligence is equal to or less than Ds


negligence. As long as P is less than 51% negligent, she will recover.

CHAPTER 11 COMPARATIVE RESPONSIBILITY IN


MULTIPARTY LITIGATION
CONTRIBUTION AND INDEMNITY

BROCHNER v. WESTERN INSURANCE COMPANY


Supreme Court of Colorado, 1986

44
724 P.2d 1293

Procedural History- Trial Court, in an indemnity action to recover monies paid in a settlement to
Ms. Cortez, awarded $150,000 to Western Insurance Company for indemnity as well as $10,000
for expenses incurred as a result of the lawsuit. Appellate Division affirmed.

Facts- Dr. Brochner, who was granted staff privileges by the Community Health Association to
perform craniotomies at Boulder Community Hospital, was sanctioned by the hospital several
times. The Hospital required that Dr. Brochner to acquire consultations before performing
craniotomies, as it appeared as though he was performing unneeded surgeries. In November of
1968, Brochner performed a craniotomy on Esther Cortez which resulted in injury to Cortez.
Cortez brought an action against the hospital and Dr. Brochner. The two claims were
subrogated. The Hospital settled the claim for $150,000. Western sought indemnity from Dr.
Brochner on the grounds that his negligence was the primary cause of Cortezs injury and the
Hospitals negligence was passive and secondary. The trial court awarded Western their
requests.

Issue(s)- Did the trial court and appellate division err in ruling that Western Insurance Company
was entitled to receive the full amount of the settlement as the Hospitals negligence had only
been secondary and passive to that of Dr. Brochner?

Holding- Yes. A tortfeasor may no longer be unfairly forced to pay all or a disproportionate share
of damages suffered by an injured party as the result of negligent conduct by two or more joint
tortfeasors. Because both Brochner and the hospital settled with Cortez, neither is entitled to
contribution from the other.

Rule(s)- Joint tortfeasors are now subject to contribution among themselves based upon their
relative degrees of fault. This new provision extends the principle that liability for negligence
should be based on proportionate fault.

Analysis-

- As a joint tortfeasor, the hospital has no right to seek indemnity from Brochner; its sole
remedy lies in contribution pursuant to the terms of the act which provides:

o When a release or a covenant not to sue or not to enforce judgment is given in good
faith to one or two or more persons liable in tort for the same injury or the same
wrongful death:

It discharges the tortfeasor to whom it is given from all liability for


contribution to any other tortfeasor.

Disposition- Because both Brochner and the hospital settled with Cortex, neither is entitled to
contribution from the other

Notes-
45
- Contribution and Indemnity

o Indemnity- the right to full reimbursement for damages paid by one party from
another

Examples

Vicarious liability- employer/employee

Contractual indemnity

Legal indemnity- seller-manufacturer

o Contribution- the right to obtain a partial reimbursement of damages paid by one


party against another

The reimbursement is by apportionment between the party who has paid and
another party who has some portion of liability

Many jurisdictions with comparative fault apportion by degree of


responsibility for the injury

- Abolishing Joint and Several Liability:

o A significant number of states have abolished joint and several liability, so that
there is no need for contribution. Each party bears his or her own share of the
responsibility.

o MS and LA- no longer have joint and several liability for negligence based actions

o However, joint and several liability has been retained for acts committed with
specific and wrongful intent or for conscious and deliberate wrongs.

o Contribution is retained for cases involving such concerts of action

- Bochner v. Western Insurance Co.

o A negligent party defendant may not recover full indemnity from another negligent
party (joint tortfeasor), but may only seek contribution in proportion to the fault of
the joint tortfeasor.

o Under existing CO law in effect before legislative abolishment of joint and several
liability

o The court abolished the active/passive negligence distinction for purposes of


indemnity because of the adoption of comparative fault

o A good faith settlement of the plaintiffs claim by one tortfeasor discharges that
tortfeasor from all liability and bars a claim from a joint tortfeasor for contribution

JOINT AND SEVERAL LIABILITY

KAEO v. DAVIS
Supreme Court of Hawaii, 1986

46
719 P.2d 387

Procedural History- Kidos guardian sued Davis and the City of Honolulu. The Citys alleged
negligence was failure to maintain the road in a safe condition. The jury found that Davis was
99% negligent and the City was 1% negligent. It also found that P suffered $725k in damages. A
judgment holding the City and Davis jointly and severally liable for the damages was entered.
City appealed.

Facts- Kido and Davis were drinking at a bar, and then went for a drive. Davis, who had been
drinking, was unable to keep the car on the winding road and hit a utility pole. Kido was severely
injured.

Issue(s)- Did the trial court err in excluding all evidence of Davis alcohol consumption? Did the
trial court err in refusing to give a jury instruction that would have apprised the jury of the
possible legal consequences of its verdict on the negligence attributable to each putative joint
tortfeasor?

Holding- Yes. The trial court erred in refusing to admit evidence of Davis alcohol consumption.
The trial court should have informed the jury of the possible legal consequence of a verdict
apportioning negligence among the joint tortfeasors

Rule(s)- The trial court, if requested and when appropriate, should inform the jury of the possible
legal consequence of a verdict apportioning negligence among joint tortfeasors.

Analysis- Wisconsin uses the Blindfold Rule, whereby the jury is not instructed on the effect that
their verdict will have as far as which defendant may ultimately be held responsible for the entire
sum. It is not the function of the jury in a case between private parties on the determination of
comparative negligence to be influenced by sympathy for either party, nor should it attempt to
manipulate the apportionment of negligence to achieve a result that may seem socially
desirable.

Disposition- Reversed and remanded.

Notes- Joint and Several Liability

- A jury should be informed, when requested by a party, of the legal effect of apportionment
of liability among joint tortfeasors

47
BROWN v. KEILL
Supreme Court of Kansas, 1978

580 P.2d 867

Procedural History- This is an appeal from a judgment for damage to Ps car resulting in a two car
accident. The trial court found: (1) that P was guilty of no negligence; (2) the son was responsible
for 90% of the causal negligence; (3) the D was responsible for 10% of the causal negligence; (4)
P sustained total damages in the amount of $5,423.00; and (5) P was entitled to recover $542.30
in damages from D. P appealed

Facts- P, Brown, allowed his son to drive his Jag, and got into a car accident with D, Keill. The
reasonable cost of repair was $5,423.00. D settled her claim against the son out of court. P then
sued to recover his property loss. D did not seek to have the son joined as an additional party,
and did not file a cross or counter claim against him.

Issue(s)- (1)Did the trial court err in refusing to apply the rule of joint and several liability of joint
tortfeasors in the state of Kansas under a comparative negligence statute? (2) Additionally, is the
causal negligence or fault of all parties to a collision or occurrence giving rise to Ps claim in a
comparative negligence action to be considered even though one of said parties is not served
with process or joined as a formal party to the action?

Holding- (1)No Trial court did not err. The concept of joint and several liability between joint
tortfeasors previously existing in Kansas no longer applies in comparative negligence actions,
due to K.S.A. 60-258(a). Yes, Although D and P both had an opportunity to bring in the third party
by way of motion, they did not take advantage of this. There is nothing in the particular statute
which specifically requires P to bring his action or file a claim against any particular person or
group of persons.

Rules(s)-

- Absent evidence of a joint venture, agency or circumstances giving rise to vicarious


liability, the negligence of a bailee of a vehicle is not imputable to the bailor in an action
by the bailor against a third party for damage to the bailed vehicle.

- The purpose and legislative intent of K.S.A. 60-258a was to impose individual liability for
damages based on the proportionate fault of all parties to the occurrence which gave rise
to the injuries and damages even though one or more parties cannot be joined formally as
a litigant or be held responsible for his or her proportionate fault.

Analysis-

48
Disposition- Affirmed

Notes-

- Absent joint venture, agency, or other special circumstances, bailor is not liable for torts of
bailee.

- Does KA retain joint and several liability?

o No

o Note p.444 rule of statutory interpretation (construction, purpose, pari material,


etc.)

- Is the negligence of a non-party joint tortfeasor considered in the allocation of


responsibility for the plaintiffs injuries?

o Yes. Any party does this mean parties to the litigation or participants to the
tort

- Effect of joint and several liability:

o Each D is liable for the full amount of damages apportioned to the Ds, regardless of
the degree of fault of each individual D

o Insolvency- who bears the risk of the insolvent joint tortfeasor? The other remaining
solvent Ds will absorb an insolvent party.

VARELA v. AMERICAN PETROFINA COMPANY OF TEXAS, INC


Supreme Court of Texas, 1983

658 S.W.2d 561

Procedural History- This is an appeal from the third-party negligence action brought by an
employee covered by workers comp. Judgment was rendered for the amount of damages found
by the jury reduced by the proportion of negligence of both the employee and the employer. The
court of appeals affirmed

Facts- Varela was employed by Hydrocarbon, who was performing a turnaround on a fluid
catalytic cracking unit owned and operated by Petrofina. During the course of performing the job,
Varela was injured due to a premises defect. After settling his workers comp claim, Valera sued
Petrofina for the damages resulting from his injuries. The jury apportioned the negligence of the
parties as follows: Varela 15%, Hydrocarbon 42%, and Petrofina 43%. The jury further found
damages in the amount of $606,800. The trial court rendered judgment for 43% of the total
damages.

49
Issue(s)- Did the trial court err in considering the negligence of the employer in a third-party
negligence action brought by an employee arising out of an accidental injury covered under
workers compensation insurance?

Holding- Yes. The injured employee may seek recovery from a 3rd party whose negligence
contributed to the injury. In the event of recovery, the negligent 3rd party is barred from seeking
contribution or indemnity from the employer, and the compensation carrier is entitled to
reimbursement for all compensation and medical expenses paid.

-Since the Work Comp Act precluded P from recovering from the employer for common law
negligence, Petrofina has no claim for contribution from employer.

Rule(s)- Where the Ds negligence is equal to or greater than that of the employee, the employee
shall recover the total amount of damages as found by the jury diminished only in proportion to
the amount of the negligence attributed to the employee.

Analysis-

Disposition- Reversed and remanded. Judgment is rendered for Varela in the amount of $515,780
plus interest from the date of judgment.

Notes-

- Jurisdictions that have abolished joint and several liability have retained it in intentional
torts

- Vicarious liability is different from Joint and Several liability. Employer is responsible for
100% of damages for employee acting in nature and scope of employment. Abolition of
joint and several liability has nothing to do with vicarious liability.

- Whose fault should be considered in apportioning responsibility?

o Non parties

o Immune persons

Employers under workers comp

Governmental immunities

Family immunities

o Phantom defendants (non-parties that are alleged to be liability in some capacity)

- May a jury consider the negligence of and assign

- If not how is fault otherwise attributable to the employer allocated?

o To the non-employer tortfeasor

50
- Note 1

- Note 2

- Note 3

- MS by recent legislature allows employers negligence to be considered in allocating


percentage of responsibility for each party involved in cause the injury

- MS allows consideration of fault of any actor who had a causal role

Partial Settlements

MCDERMOTT, INC. v. AMCLYDE


Supreme Court of the United States, 1994

511 U.S. 202

Procedural History- Jury found damages of 2.1 million. Responsibility was allocated as follows:
32% AmClyde, 38% River Don, and 30% jointly to McDermott and the sling defendants. Court
denied motion to reduce the judgment by the $1 million dollar settlement, and entered judgment
against AmClyde for $672,000 and against River Don $798,000. Court of Appeals held that a
contractual provision precluded any recovery against AmClyde and that the trial judge
improperly denied a pro tanto settlement agreement. The Appellate Court reversed the
judgment award and reduced it as follows: First, it reduced the 30% negligence on the part of
McDermott. This came to $1.47 million. Next, it deducted the 1 million received in settlement to
reach $470,000.

Facts- McDermott, petitioner, bought a 5,000 ton crane from AmClyde. When Petitioner first
used the crane to attempt to move an oil and gas production platform, a prong off the cranes
main hook broke, causing massive damage to the deck and to the crane itself. The malfunction
may have been caused by petitioners negligent operation of the crane, by AmClydes faulty
design or construction, by a defect in the hook supplied by River Don, or by one or more of 3
other companies that supplied the supporting steel slings. On the eve of trial, Petitioner entered
into a $1 million settlement dismissing the sling defendants. They released them from all
liability for either deck or crane damages, and agreed to indemnify them against any
contribution action. McDermott assumed any liability the slings may have had in causing the
accident.

Issue(s)- Should the liability of the non-settling defendants be calculated with reference to the
jurys allocation of proportionate responsibility? Or should the Court give the non-settling Ds a
credit for the dollar amount of the settlement?

Holding- The proportionate share rule announced in this opinion applies when there has been a
settlement. In such cases, the Ps recovery against the settling D has been limited not by
outside forces, but by its own agreement to settle. There is no reason to allocate any shortfall to
the other defendants, who were not parties to the settlement. Just as the other defendants are
51
not entitled to a reduction in liability when P negotiates a generous settlement, they are not
required to shoulder disproportionate liability when the P negotiates a meager one.

Rule(s)- It is generally agreed that when a P settles with one of several joint tortfeasors, the non-
settling Ds are entitled to a credit for that settlement.

3 alternatives outline by the American Law Institute:

1. Pro Tanto set off with right to contribution against the settling D-The money paid
extinguishes any claim that the injured party has against the party released and the amount of
his remaining claim against the other tortfeasor is reached by crediting the amount received; but
the transaction does not affect a claim for contribution by another tortfeasor who has paid more
than his equitable share of the obligation.

-This discourages settlement, because settlement can only disadvantage the settling
defendant.

2. Pro tanto rule without contribution-The money paid extinguishes both any claims on the
part of the injured party and any claim for contribution by another tortfeasor who has paid more
than his equitable share of the obligation and seeks contribution. As in Alternative #1, the
amount of the injured partys claim against the other tortfeasors is calculated by subtracting the
amount of the settlement from the Ps damages.

-The pro tanto approach, even when supplemented with good-faith hearings, is likely to
lead to inequitable apportionments of liability.

-By disadvantaging the party that spurns settlement offers, the pro tanto rule puts
pressure on all defendants to settle. The additional incentive provided by the pro tanto rule
comes at too high a price in unfairness.

3. Settling tortfeasor and the proportionate share approach-The money paid extinguishes
any claim that the injured party has against the released tortfeasor and also diminishes the claim
that the injured party has against other tortfeasors by the amount of the equitable share of the
obligation of the released tortfeasor.

-The first 2 alternatives involve the kind of pro tanto credit that respondents urge the
Court to adopt. The only difference is the recognition of a right of contribution against a settling
defendant in the first but not the second. The third alternative involves a credit for the settling
defendants proportionate share of responsibility for the total obligation. The Court uses this
approach here.

One Satisfaction Rule- This is no longer good law. Historically, this rule barred a P from
litigating against another joint tortfeasor, if he had settled with and released another.

Analysis- The proportionate share rule announced in this opinion applies when there has been a
settlement. In such cases, the Ps recovery against the settling D has been limited not by
52
outside forces, but by its own agreement to settle. There is no reason to allocate any shortfall to
the other defendants, who were not parties to the settlement. Just as the other defendants are
not entitled to a reduction in liability when P negotiates a generous settlement, they are not
required to shoulder disproportionate liability when the P negotiates a meager one.

Disposition- Reversed court of appeals decision and remanded

Notes-

- What happens if joint and several liability and a D settles for more than his fair share?

o Did the release include or exclude the other D? If settlement released other Ds
also, then contribution allowed (Restatement 3d).

Common law rule- a release to any of the Ds- you release all of them

Modified- if you release A, but reserve the right to sue B and C, then you can
release A and still sue B and C

Typically the release is a release of the party of who is on the release only
everyone else is still liable if you have a claim against them.

- In most jurisdictions the settling D who pays (at least in good faith) less than
what turns out to be his fair share is completely released and no claim for
contribution may be made against the settling D.

- Partial Settlements

o Is there joint and several liability?

If not, then settlement by on D will not affect the liability of a joint tortfeasor.

The nonsettling defendant would pay in the amount that

However, where there is joint and several

Several methods have been used:

o Pro rata-(equal division among the Ds without regard to % of


responsibility for the injury)

o Pro tanto- a dollar for dollar reduction allowed against the award

o Proportionate: more modern trend- the award is reduced by the


% of responsibility attributed to the settling defendant

- McDermott

o Issue:

In an admiralty case, should the amount of settlement with settling Ds who


pays more than its fair share of the damages

o Holding

53
Pro-tanto

Proportionate- Court goes with this approach. Ps claim against the settling D
is released and the claim

APPLICATION OF COMPARATIVE RESPONSIBILITY TO CAUSES OF


ACTION OTHER THAN NEGLIGENCE

BONPUA v. FAGAN
Superior Court of NJ, Appellate Division, 1992

Page 461

Procedural History- Trial Court partially granted Ps Motion for Summary judgment on the
grounds that Ds conviction for aggravated assault conclusively established that he committed
an intentional tort upon P, and that under the Comparative Negligence Act, any negligence or
other fault of P would not bar or reduce his claim. D appeals.

Facts- P and D were involved in a physical altercation in the parking lot behind Cammaranos Bar
in Long Beach on March 26, 1987. This fight stemmed from an altercation in the bar in which P
called D a faggot. D claims that he walked over to Ps car and P began hitting him, and he was
forced to defend himself. Both P and D have suffered injuries. D was convicted of aggravated
assault and sentenced to a 7 year prison sentence.

Issue(s)- Did the trial court err in partially granting Ps motion for summary judgment, striking
defendants affirmative defense of comparative negligence?

Holding- Yes. The Comparative Negligence Act requires the tries of fact to apportion the fault of
all culpable parties, regardless of whether their conduct was negligent or intentional.
Defendants criminal conviction does not preclude him from relying upon the Comparative
Negligence Act as a defense to Ps claim.

Rule(s)- The Comparative Negligence Act requires the tries of fact to apportion the fault of all
culpable parties, regardless of whether their conduct was negligent or intentional.

-A criminal conviction does not place a limitation on the Act

54
Reasoning/Analysis- The criminal jury could have returned a guilty verdict even though it
believed P initiated the altercation if it found that D responded to Ps verbal abuse by assaulting
him or used excessive force in repelling Ps aggression. Thus, a finding that both P and D were
negligent in causing the injury to P would not be inconsistent with the criminal jurys finding.

Disposition- Reverse and remanded.

Notes

- Apportionment of fault when fault claim is other than negligence

- Should comparative fault be used in allocation of fault involving intentional torts?

- Should comparative fault be used in allocation of responsibility involving strict liability


claims (esp. in products)? Tendency is for courts to use comparative fault in reducing the
claim against the strictly liable party.

- Cases involving a mis of intentional torts and negligence

- Apportionment by causation and fault in the same case. Mostly unresolved by the courts
(Section F. on 466)

55
CHAPTER 12 - IMMUNITIES
A. GOVERNMENT IMMUNITY

1. STATE AND LOCAL GOVERNMENT

HICKS v. STATE
Supreme Court of New Mexico, 1975

Page 470

Procedural History- This is an appeal from an order of the DC dismissing Ps complaint on the
basis of sovereign immunity. Suit was originally brought to recover damages for the wrongful
death of Ps wife and daughter for the negligence of the State Highway Dept., when a school bus
collided with a cattle truck on a bridge built and maintained by the State Highway Dept. D filed a
MTD which was granted by the DC. The DC stated that the doctrine of sovereign immunity was a
long-standing common law principle which could now be changed only by legislation.

Facts-

Issue(s)- Did the DC err in granting Ds MTD based on the common law principle of sovereign
immunity?

Holding- Yes. The doctrine of sovereign immunity has always been a judicial creation without
statutory codification. Just because a court made rule has been in effect for many years does
not render it invulnerable to judicial attack once it reaches a point of obsolescence. Sovereign
immunity may no longer be interposed as a defense by the state, or any of its political
subdivisions, in tort actions. In addition, its continuance is causing a great degree of injustice.

Rule(s)- Common law sovereign immunity may no longer be interposed as a defense by the
state, or any of its political subdivisions, in tort actions. It can no longer be justified by existing
circumstances and has long been devoid of any valid justification.

- Ayala v. Philie Board of Pub. Education- no reasons exist for continuing to adhere to the
doctrine of governmental immunity. Whatever may have been the basis for the inception
of the doctrine, it is clear that no public policy considerations presently justify its
retention.

Analysis- The argument has been made that eliminating sovereign immunity will result in a
massive financial burden on the state. However, this is a weak argument because insurance
should cure that problem. In addition, placing the financial burden on the state would be fair,
because they can afford it, rather than placing the financial burden on the individual alone.
56
Disposition- Reversed

Motion on Rehearing-

Issue: Whether this decision should apply (1) only to cases arising in the future; (2) to cases
arising in the future and to the case at bar, or (3) to cases arising in the future, to the case at bar
and to all similar pending actions.

Holding- Should only apply to cases arising in the future so that the government may have ample
notice and obtain liability insurance.

Notes-

- Immunities

o Immunity is a defense that bars the Ps claim even if the D would otherwise be
liable and is based on the status of the D or the legal relationship between the P
and D.

o Immunity may take the form of an affirmative defense or as a preclusion of subject


matter jurisdiction (governmental).

o Immunity is in effect the freedoms liability for conduct that otherwise create
liability.

o Types:

Sovereign immunity/ governmental immunity

Charitable immunity- old rule- charitable institutions had immunity under


common law because without this immunity, these institutions wouldnt offer
their services for the public good. Has largely been abolished or severely
restricted (almost no longer exists).

Family immunities- an area which has been diminishing in importance. Ex-


husbands and wives could not sue each other in common law, because the
husband and wife had a single identity. Because the wifes identity was
subsumed into the husbands, it wouldnt make sense for the husband to sue
himself.

Employer immunity in tort (Workers Comp.)

- Hicks v. State

o Should New Mexico retain the doctrine of sovereign immunity in tort claims against
the state?

NO!!! Not any more

57
o Policies behind sovereign immunity

Govt can do its job properly as opposed to being distracted

Public inconvenience

Keeps taxes lower

Cost of insurance is too much

o Policies for abolition

Individual shouldnt bear the burden alone

Govt can get insurance

Matter of fairness, equity, and justice

Public should bear the loss of governmental mistakes as opposed to the


injured individual bear the entire cost/injury alone

Common law rule created by the court, that the court has the power to
abolish if it so chooses (if it is a common law rule, then it seems as though,
the judges have the authority to change it. The other philosophy is that major
changes in the law should come from legislatures that are elected to change
it by the democracy.)

Spreading the costs of the injury so that no one individual is catastrophically


affected through no fault of his own.

o Proprietary v. governmental functions of municipal (city, county and other local


government units)

Proprietary function- the govt is reliable.

o Note 1

Most states have at least partially abolished sovereign immunity

o Note 2

o Note 3

o Note 4 federalism and the federal constitution

Cant sue a state in federal court in tort

Generally a matter of state law and fed courts will not take jurisdiction except
in the case of civil rights cases

58
- Federal Government Immunity governed by the Federal Tort Claims Act (FTCA)

o FTCA-

Involves claims against the federal govt, agencies or employees

o Jurisdiction

Exclusively in the federal district courts for money damages by torts of


federal employees acting in the scope of employment

o Time Limits

2 years to file a claim with the agency (maximum time)

Must sue within 6 months of final denial of claim (file suit in federal court)

If agency fails to act on timely claim, claimant may file suit after 6 months
(but claim must be timely)

o No jury trial against the U.S.

o If U.S. is liable, it is liable for the same damages that would be available against a
private person, except:

No punitive damages

No pre-judgment interest

o In wrongful death claims, if only damages under applicable state law are punitive,
U.S. is liable for compensatory damages instead (measured by the pecuniary
damages)

Compensatory instead of punitive

o Exceptions:

No liability to the US for torts that:

Are based on employees exercising due care in executing statutes or


regulations (without regard to validity of the statute of regulation)

Or arise from performance or not of discretionary functions

No claims for assault, battery, false imprisonment, malicious prosecution,


abuse of process, defamation, misrepresentation, deceit or interference with
contracts

o Exceptions to the exceptions:

Assault, battery, false imprisonment, false arrest, abuse of process, or


malicious prosecution with regard to actions or omissions by federal law
enforcement or investigative officers (with law enforcement powers)

o No claims arising from combat activities during war time

o No claims arising in foreign countries

59
o No claims for Treasury fiscal operations or monetary regulation

60
2. FEDERAL GOVERNMENT

a. FEDERAL TORT CLAIMS ACT

UNITED STATES v. GAUBERT


SCOTUS, 1991

Page 478

Procedural History- Gaubert filed an action in the DC against the US under the Federal Tort
Claims Act (FTCA), seeking $100 million damages for the lost value of his shares and for the
property forfeited under his personal guarantee on the ground that the FHLBB and Dallas division
were negligent in carrying out their supervisory activities. The DC granted Ds MTD for lack of
SMJ on the ground that the regulators actions fell within the discretionary function exception to
the FTCA. Relying on Indian Towing Co. v. U.S., the Court of Appeals found that the claims
concerning the regulators activities after they assumed a supervisory role in IASAs day-to-day
affairs were not policy decisions, which fall within the exception, but were operational
actions, which it believed were not within the exception.

Facts- a federal statute authorized the Federal Home Loan Bank Board (FHLBB) to proscribe rules
and regulations governing federal savings and loan associations. During the savings and loan
crisis of the FHLBB and the branch in Dallas oversaw certain operations of Independent American
Savings Association (IASA), but instituted no formal action against the institutions. At the
FHLBBs request, respondent Gaubert removed himself from IASAs management and posted $25
million as security for his personal guarantee that IASAs net worth would exceed regulatory
minimums. Later the regulators threatened to close IASA unless all management resigned. They
did so and were then replaced. Thereafter, FHLBB-Dallas became more involved in IASAs
everyday business activity. Although IASA was thought to be financially stable while Guabert
managed it, after a few months the new directors announced that IASA has a substantial
negative net worth, and the Federal Savings and Loan Insurance Corporation assumed
receivership of the institution.

Issue(s)- Did the Court of Appeals err when it reversed in part on the ground that the claims
concerning the regulators activities after they assumed a supervisory role in IASAs day-to-day
affairs were not policy decisions, which fall within the discretionary function exception, but
were operational actions, which it believed were not within the exception?

Holding- Yes. A discretionary act is one that involves choice or judgment; there is nothing in that
description that refers exclusively to policymaking or planning functions.

Rule(s)- The exception covers only acts that are discretionary in nature, acts that involve an
element of judgment or choice, and it is the nature of the conduct, rather than the status of
the actor that governs whether the exception applies.

-The requirement of judgment or choice is not satisfied if a federal statute, regulation, or


policy specifically prescribes a course of action for the employee to follow, because the
employee has no rightful option but to adhere to the directive.
61
-Discretionary function exception is only intended to prevent judicial second-
guessing of legislative and administrative decisions grounded in social, economic, and
political policy through the medium of an action in tort.

The exception only protects governmental actions and decisions based on


considerations of public policy.

-When established governmental policy, as expressed or implied by statute,


regulation, or agency guidelines, allows a Government agent to exercise discretion, it
must be presumed that the agents acts are grounded in policy when exercising that
discretion.

Analysis-

Disposition- Reversed and remanded

Case Notes-

- Note 3

- Note 2

- Note 1

- Note 4-5

b. JUDICIALLY CREATED IMMUNITY

UNITED STATES v. JOHNSON


SCOTUS, 1987

Page 486

Procedural History- Respondent, Johnsons wife, sought damages from the US on the ground that
the FAA flight controllers negligently caused her husbands death. The DC dismissed the
complaint relying on the Feres decision. The Court of Appeals for the 11th Circuit reversed. The
court found that the evolution of the doctrine since the Feres decision warranted a qualification
of the original holding according to the status of the alleged tortfeasor. The court found
absolutely no hint that the conduct of any alleged tortfeasor even remotely connected to the
military will be scrutinized if this case proceeds to trial. Accordingly it found that Feres did not
bar respondents suit.

Facts- Johnson was a pilot for the US Coast Guard. Johnsons Coast Guard Station received a
distress call from a boat lost in the area. Johnson and a crew of several other Coast Guard
62
members were dispatched to search for the vessel. Inclement weather decreased the visibility,
so Johnson requested radar assistance from the Federal Aviation Administration (FAA), a civilian
agency of the Federal Govt. the FAA controllers assumed positive radar control over the
helicopter. Shortly thereafter, the helicopter crashed into the side of a mountain. Johnson, along
with all of the other crew members, was killed in the crash.

Issue(s)- Did the Court of Appeals err in concluding that the Feres Doctrine did not bar recovery
when negligence is alleged on the part of a Federal Government Employee who is not a member
of the military?

Holding- Yes. There is no doubt that Johnson was killed while performing a rescue mission on the
high seas, his injuries arose directly from that mission; his wife continues to receive death
benefits; because Johnson was acting pursuant to the standard of operating procedures of the
Coast Guard, the potential that this suit could implicate military discipline is substantial. The
Feres Doctrine should have been applied.

Rule(s)- Feres Fatual Paradigm- Exists when a service member alleges negligence on the part
of another member of the military. When this is present, the question becomes, Did the injury
arise out of or during the course of an activity incident to service? The government is not
liable under the FTCA for injuries of servicemen where the injuries arise out of or are
in the course of activity incident to service.

3 broad rationales underlying the Feres decision:

1. The relationship between the Government and members of its armed forces is distinctively
federal in character;

2. Generous statutory disability and death benefits are provided as a remedy;

3. These are the types of claims that, if generally permitted, would involve the judiciary in
sensitive military affairs at the expense of military discipline and effectiveness.

Analysis- Dissent says- READ THE STATUTE AS IT WAS WRITTEN!! This would render the federal
government liable to any individual, servicemen included, for the negligence of a government
employee.

Disposition- Reversed and remanded

Notes-

- Issue: Does a military employee of the U.S. have FTCA claim against the government for
negligent harm caused by a civilian federal employee, when the injury occurs while the
military employee is involved in a service activity?

- 11th Circuit held in this case: Feres doctrine applies only to bar claims of military personnel
when injured by other military personal, not when injured by civilian employees of the
federal government.
63
- Feres doctrine, according to the S.Ct., in this case: bars tort liability for the govt in any
case in which service member is injured arising out od or in the course of activity incident
to service, whether the injury is casued by civilian or military government

- Rationale:

o Military employment is by its nature risky. Site of injury should not affect govts
liability

o Service related injuries are subject to statutory benefits, so a remedy is available

- Congress has not amended the statute providing for benefits to injured service personnel
since the Feres doctrine was declared.

- Congress was unlikely to have intended both tort recovery and recovery under the
Veterans Benefit Act

- To permit torts liability in such cases would involve the judiciary in matters best left to
military. Judicial involvement could negatively affect military discipline and effectiveness.

- Scalia: The statutory language is plain. Congress did not create an exception governing
military personnel except in combat situations.

o No presumption as to Congress intent in passing in the FTCA should be drawn from


the lack of amendment by subsequent Congressess

o None of the supporting rationales offered by the majority withstand scrutiny

3. IMMUNITY OF GOVERNMENTAL OFFICERS AND EMPLOYEES


- Federal Liability Reform and Tort Compensation Act of 1988.

- Absent violation of Constitution or federal stature, federal employees are immune upon
the employees request, the attorney general certifies that the employee was acting within
the scope of employ

- State Immunity

o Absolute immunity generally provided to judges, prosecutors, and high-ranking


executives, acting within powers of their offices.

o Qualified immunity is usually provided to other employees for torts committed


within the scope of their employment if they act in good faith belief that the acts
are lawful

o Generally state or local governments cover all or part of judgments against


employees acting within scope.

4. THE PUBLIC DUTY DOCTRINE

RISS v. CITY OF NEW YORK


New York Court of Appeals, 1968

Page 495
64
Procedural History- Trial court dismissed the complaint. Appellate division affirmed.

Facts- Linda Riss was stalked for over 6 months by a psycho, Burton Pugach. He repeatedly
threatened to kill or injure her if she did not go out with him. In fear for her life, she called the
police, but they did nothing. Linda subsequently was engaged to another man. At their
engagement party, the psycho called her and told her it was her last chance. She then called
the police again and was refused assistance. The next day Pugach hired a thug to throw lye in
Lindas face. She was blind in one eye, lost substantial vision in the other, and her face was
scarred. After the assault, she was afforded round-the-clock protection for the next 3.5 years
from the police.

Issue(s)- Did the appellate division err in affirming the trial courts decision to dismiss the
complaint because the police department owed the public no duty to prevent this kind of injury?

Holding- No. It would be impractical for the Court to carve out an area of tort liability for police
protection to members of the public. Police resources are limited by the resources of the
community and by a legislative-executive decision as to how those resources may be deployed.

Rule(s)- Police officers cant be held liable to members of the public who are injured by criminal
third parties. Before such extension of responsibilities should be dictated by the indirect
imposition of tort liabilities, there should be a legislative determination that that should be the
scope of public responsibility.

-However, this immunity would not apply if the police authorities undertook responsibilities
to particular members of the public and then exposed them, without adequate protection, to the
risks which then materialize into actual loss.

Analysis-

Disposition- Affirmed

Notes-

- Public duty doctrine

o No duty to rescue

o No general duty to protect third parties from harm by third persons

o No duties to protect specific individuals from harm where there is a duty owed to
the general public

65
o Some state limit the public duty doctrine to police protection

Riss:

Local govt does not owe a duty to one individual to protect them from harm caused by
criminal wrongdoers.

Court distinguished this kind of govt activity from others activities such as duties that arise
in the course of operating public

The court did not feel that it was appropriate to become involved with the choices of eth
allocation of resources by local govt. The level of police protection was better left to leg
decisions. Court is not competent to make such determinations.

However, if the police authorities undertake responsibilities to specific members of the public,
the govt can be held for negligence in failing to act reasonably performing such undertakings.

B. FAMILY IMMUNITIES

PRICE v. PRICE
Supreme Court of Texas, 1987.

732 S.W.2d 316

Procedural History- Summary Judgment was granted for defendant, Duane Price, in this
negligence action. Court of Appeals affirmed the judgment. Kimberly Price appeals.

Facts- Kimberly Price was injured in July of 1983 when the motorcycle of which she was a
passenger, driven by her future husband, Duane Price, collided with a truck. Kimberly married
Duane 6 months later and brought suit against Duane and the other truck driver, alleging that
their negligence combined to cause her injury. The driver of the truck and his employer settled
out of court, but the court issued summary judgment for Duane, relying on the doctrine that
one spouse could not sue another for negligence (Interspousal Immunity).

Issue(s)- Did the Court of Appeals err in ruling that the doctrine of interspousal immunity barred
Kimberly Price from recovering against her husband, Duane Price?

66
Holding- Yes. The interspousal immunity doctrine is no longer good law and his abolished. The
Married Women Acts gave wives the right to own, acquire and dispose of property, to contract,
and to have the right to sue in regards to their own property in contracts. The Common law
holding that a husband and wife are one person is no longer valid. It is difficult to fathom
how denying a forum for the redress of any wrong could be said to encourage domestic
tranquility. The Doctrine of Spousal Immunity is abolished completely as to any cause of action.

Rule(s)- The common law rule for interspousal immunity is no longer good law. The Doctrine of
Spousal Immunity is abolished completely as to any cause of action.

Traditional justifications for the rule (of spousal immunity): legal unity, husbands status, spousal
harmony, and collusive suits

Disposition- Reversed and Remanded.

BROADBENT v. BROADBENT
Supreme Court of Arizona, 1995.

Page 503

Procedural History- Trial Court granted the mother, Laura Broadbents, Motion for Summary
Judgment on the grounds that the parental immunity doctrine applied to the facts of this case.

Facts- Christopher Broadbent, Plaintiff/Appellant, and his mother, Laura Broadbent,


Defendant/Appallee, went swimming at their residence on April 13, 1984. Christopher was
wearing his floaties, as he was only 2 and years old and did not know how to swim. Laura left
Christopher by the side of the pool to go answer the telephone even though she had seen him
remove his floaties. Laura talked on the phone for 5-10 minutes before she noticed that she
could not see Christopher from where she was talking. She found him floating in the deep end of
the pool and began CPR on him. The paramedics arrived shortly thereafter and Christopher was
finally revived at the hospital. Due to the long length of oxygen depravation, Christopher
suffered sever brain damage and now has lost his motor skills and voluntary movement.

Issue(s)- Did the Trial Court err in granting summary judgment for the Defendant under the
parental immunity doctrine?

Holding- Yes. The trier of fact must apply the Reasonable Parent Standard. The trier of fact
must decide whether the mother in leaving the 2 and year old child unattended next to the
swimming pool is reasonable or prudent.

67
Rule(s)- Exceptions to Parental Immunity: if the parent is acting outside his parental role and
within the scope of employment; if the parent acts willfully, wantonly, or recklessly; if the child is
emancipated; if the child or parent dies; if a third party is liable for the tort, then the immunity of
the parent does not protect that third party; and if the tortfeasor is standing in loco parentis,
such as a grandparent, foster parent, or teacher, the immunity does not apply.

Reasons in Support of Parental Immunity:

(1) Suing ones parents would disturb domestic tranquility;


(2) Suing ones parents would create a danger of fraud and collusion;
(3) Awarding damages to the child would deplete family resources;
(4) Awarding damages to the child would benefit the parent if the child predeceases
the parent and the parent inherits the childs damages;
(5) Suing ones parents would interfere with parental care, discipline, and control.
Reasonable Parent Standard for Parent-Child Suits- A parents conduct is judged by
whether that parents conduct comported with that of a reasonable and prudent
parent in a similar situation!!! A parent is not immune from liability for tortious conduct
directed toward his child solely by reason of that relationship. And, a parent is not liable for an
act or omission that injured his child if the parent acted as a reasonable and prudent parent in
the situation would.

-The Reasonable Parent standard attempts to fashion an objective standard that does not
result in second-guessing parent in the management of their family affairs.

-Overrules Sandoval, which created the duty to the world at large versus duty to
the child alone distinction. Also overrules the Goller Standard and Test.

Reasoning/Analysis-

Disposition- Reversed and Remanded

Parental Immunity Notes

Under what circumstances should a parent be held liable in tort to a child for injury caused by
parents fault?

American common law rule parents are immune from tort suits by minor or emancipated
children.

Policy reasons: family harmony, parental discipline, family resources for other family members
(spec. other children), and undue intrusion by the states into matters of the family. Parental
inheritance by wrongdoing parent. (so parent gets to keep the money in the 1st place, assuming
the parent wouldnt spend the money on something else. (see above)

68
Rule eroded over time no immunity for intentional torts, no immunity for reckless, willful torts,
no immunity in auto accidents, then no immunity when duty is one owed generally to others.
This continued to the point that parents are liable exept with reference to matters of legal
obligations. Continued to the point of where the law was at the time of Broadbent

Broadbent case:

Parental immunity is abolished. Parents are liable in tort when they have failed to act as
reasonable parents in the situation.

-Insurance is gen available to cover liability or suit is not brought. Parents are gen the parties
who actually initiate the suit against themselves. (does create a conflict of interests)

-Likelihood of inheritance by parent is remote. Reasonable parent rule permits sufficient


discretion and flexibility to accommodate different styles of parenting.

-serious injuries are mor likely to occur in exercising parental authority when the parent acts in a
willful pr wanton manner (such as severely beating a child) and such conduct should not be
protected.

CHAPTER 13 - MEDICAL MALPRACTICE


PROFESSIONAL STANDARD OF CARE

MELVILLE v. SOUTHWARD
Supreme Court of CO, 1990

Page 512

Procedural History- Trial Court awarded P a $56,000 award for malpractice of a podiatrist, which
resulted in a permanent foot injury. Court of Appeals overruled. Plaintiff appeals.

Facts- In 1980, P consulted with D about an ingrown toenail. D removed the ingrown toenail and
also suggested that P have a metatarsal osteotomy to relieve discomfort. The podiatrist
performed the surgery a month later in his office. One week later, the P returned to Ds office for
a checkup. The doctor said that he did not like the looks of the foot, and provided P an additional
antibiotic. After another week, P complained that her foot was red, swollen, and painful. D told
her to increase the amount of vinegar and soak the foot more frequently. Two days later, D told
P that her wound was healing properly. The next day P noticed fluid coming out of the area near
the incision and called her family physician, who in turn referred her to Dr. Barnard, an ortho
surgeon. Barnard testified that the osteotomy procedure was below the standard of care for 2
reasons: (1) Because it was unnecessary and (2) the procedure was performed in an unsterile

69
office environment. He also testified that the post-operative standard of care was also below the
standard.

Issue(s)- Did the trial court err in allowing an orthopedic surgeon to be an expert witness as to
the degree of care owed by another specialty, a podiatrist? Was the Court of Appeals correct in
ruling that the testimony offered by Dr. Bernard with regard to the standard of care during and
after the operation was nothing more than opinion testimony?

Holding- Yes. The P failed to establish an evidentiary foundation that Dr. Bernard, by reason of
his knowledge, skill, experience, training, or education, was so substantially familiar with the
standard of care for podiatric surgery as to render his opinion testimony as well informed as that
of a podiatrist. Nor did the P establish that the standard of care for the procedure performed was
identical for both orthopedics and podiatrists.

Rule(s)- In a medical malpractice case, the P must establish that the D failed to conform to the
standard of care ordinarily possessed and exercised by members of the same school of medicine
practiced by the Defendant.

-The standard of care in a medical malpractice case is measured by whether a reasonably careful
physician of the same school of medicine as the D would have acted in the same manner as did
the D in treating and caring for the P.

-In order to establish what the controlling standard of care owed is, the P must introduce expert
testimony. The court must make 2 preliminary determinations regarding the expert testimony:
(1) Will the expert testimony assist the trier of fact in understanding the evidence or in
determining a fact in issue; (2) Is the expert witness properly qualified by knowledge, skill,
experience, training, or education to offer an opinion on the issue in question.

Reasoning/Analysis-

Disposition- Remanded to the district court for a new trial.

Notes-

Negligence: (must show all of these)

o Duty- Standard of Care (Other physicians in the same school of medicine whether
a reasonably careful physician (mentally competent person) of the same school of
medicine would have acted the same way)

o Breach-

o Cause in fact-

o Legal Cause-

o Damages
70
In medical malpractice cases the standard of care is measured by the skills ordinarily
possessed and exercised by members of the same field or school of medicine as practiced
by the defendant. (think about this as what the bas level of minimum competency
accepted within the profession, not what the skill level of the average m.d. would be)

Duty is to use reasonable care, but reasonable care is defined by reference to the
professional standard of care for the applicable profession, specialty or subspecialty of the
profession involved. (where one has a recognized profession- licensed or regulated-
have established standards of care, or rules of ethics)

Other courts:

o The standard of care is the standard of a minimally competent practitioner in that


specialty or field. (What is reasonable in that specialty?) The test is then, what
would a minimally competent or qualified doctor in that field have done under the
circumstances?

Note: Not the average doctor. (1/2 would be below that standard)

o In other words,

Expert testimony:

o Ordinarily needed to establish what the standard of care is

o The expert must be familiar with the defendants specialty (usually in the same
specialty) or be in a specialty which has substantially identical standards of care.

o Note: the experts personal opinion of what should have been done is not sufficient.
The expert must establish what the standard of care is for the specialty, not his
personal standard or preference.

There are two parts to the expert opinion needed:

o What is the Standard of care applicable to the D?

o Did the D fail to meet the applicable SOC?

P must establish both- and then must also establish cause in fact and legal cause

As to cause in fact, that often will require expert testimony as well, but is really a separate
matter.

The expert need not be in the same specialty to establish cause in fact

Professional standards in other professions:

o lawyers, accountants, architects, engineers

National v. Local standard

o Specialties- generally national

o General practices-

Most states- either modified local (i.e., same or similar communities) or


national

71
But generally can take in account availability of facilities- rural vs. city

Alternative medicine:- Note 5 page 519

o Holistic, eastern, etc.

HARNISH v. CHILDRENS HOSPITAL MEDICAL CENTER


Supreme Judicial Court of MA, 1982.

Page 520

Procedural History- A medical malpractice tribunal concluded that the Ps offer of proof was
insufficient to raise a question appropriate for judicial inquiry. The action was dismissed after P
failed to produce a bond. P appeals the dismissal.

Facts- P underwent an operation to remove a tumor in her neck. During the procedure, her
hypoglossal nerve was severed, allegedly resulting in permanent and almost total loss of her
tongue. Ps complaint charges the physicians and the hospital with failing to inform her before
surgery of the risk of loss of tongue function. Further, the complaint alleges that the loss of the
tongue function was a material and foreseeable risk, and that, had she been informed of the risk,
she would not have consented to the operation. A medical malpractice tribunal concluded that
the Ps offer of proof was insufficient to raise a question appropriate for judicial inquiry.

Issue(s)- Did the medical malpractice tribunal err in ruling that Ps offer of proof was insufficient
to raise a question appropriate for judicial inquiry?

Holding- Yes. A physicians failure to divulge in a reasonable manner to a competent


adult patient sufficient information to enable the patient to make an informed
judgment on whether to give or withhold consent to a medical or surgical procedure
constitutes professional misconduct. P must only prove that had the proper information
been provided neither she nor a reasonable person in similar circumstances would have
undergone the procedure.

Rule(s)-The Rule of Liability- The performance of a surgical procedure by a physician without


the patients consent constitutes professional misconduct.

Doctrine of Informed Consent- It is the prerogative of the patient, not the physician, to
determine the direction in which his interests lie.

-Every competent adult has the right to forgo treatment, or even cure, if it entails what for
him are intolerable consequences or risks however unwise his sense of values may be in the
eyes of the medical profession.

A physician owes to his patient the duty to disclose in a reasonable manner all
significant medical information that the physician possesses or reasonably should
possess that is material to an intelligent decision by the patient whether to undergo a
proposed procedure.

The information that a physician reasonably should possess is that information


possessed by the average qualified physician or, in the case of a specialty, by the average
qualified physician practicing that specialty.

72
The extent to which he must share that information with his patient depends upon what
information he should reasonably recognize is material to the plaintiffs decision. Materiality
may be said to be the significance of a reasonable person, in what the physicians knows or
should know is his patients position, would attach to the disclosed risk or risks in deciding
whether to submit or not submit to the treatment.

Reasoning/Analysis-

Disposition- Reverse dismissal of claim against Dr. Holmes and Dr. Mulliken, uphold the dismissal
against the hospital and Dr. Gilman.

Notes-

- Physician should disclose the material risks that may occur during the proposed
procedure

- Material risks will vary depending on the type of surgery, whether it be elective or life
saving.

- Doctor must disclose the risks that are material and inherent in the procedure, in addition
to the risks of non treatment and alternative treatments.

- Informed Consent

o Battery or Negligence

Battery if no consent at all is given and the touching is intentional and


harmful

Negligence if consent to medical treatment by a physician, but the consent is


based on inadequate disclosure of risks.

o Why have informed consent?

Protects the patients sense of self-autonomy or bodily integrity the patient


should have the right to say what should be done to him. (not to individuals
specifically, it is more general. Reasonable person standard)

Protects patients decision making

- Harnish v. Childrens Medical Center

o Physician is guilty of malpractice when the physician performs surgery without the
patients consent

o P must prove the failure of the Dr. to disclose the material facts. In addition, she
must prove that but for the Dr. telling her, she wouldnt have gone through with the
surgery.

o Then P must prove that a reasonable person would have acted the same given the
circumstances

- Policy: Patients interest

o Physician is under a duty to disclose in a reasonable manner to competent adult


patients sufficient information to enable patient to make an informed judgment as

73
to whether to undergo medical or surgical treatment. (promotes more patient
centered autonomy as opposed to the other side)- half the states- patient autonomy
standard

- Other side- what would a reasonable physician disclose to a patient as being a material
risk

- Disclosure required is: all significant medical information possessed by physician or that
he or she reasonable possesses, that is material to an intelligent decision by the patient
whether to undergo a proposed. Other half the states- physician centered standard-
more modern trend rule.

- What the physician should known in this context must be established by expert testimony
(what knowledge should be possessed in the doctors field of practice)

- Materiality determination- what information has significance in the decision making


process to one in the patients situation as known to the physician or what as it should be
known to the physician

- Materiality determination DOES NOT require expert testimony

- Factors to be disclosed: (merely an illustrative list- not exclusive or exhaustive)

o Nature of condition

o Nature and probability of risks

o Benefits expected

o Uncertainty of results

o Whether, if relevant, results are reversible

o Alternative procedures with the attendant risks and benefits

- Court adopts patient-centered approach- Harnish Case

o Reasonableness of disclosure is based on what a reasonable patient would need


to know

- About half the state have physician-centered approach

o What would a reasonable physician disclose? (or what is customarily disclosed)

Expert testimony to establish what is customary in the field

- Which approach is better?

o Patient center approach is more uncertain

o We are trying to balance the two-

- Duty, breach, cause in fact (But for test), Legal cause (was it foreseeable?)

- If the physician then breaches the duty of informed consent, patient still must prove

o Cause in fact of harm as result of undergoing the procedure and the occurrence of
that harm from an undisclosed risk that should have been disclosed.

74
- P must also prove that a reasonable person would not have undergone the procedure

- Problem: is there a range of acceptable reasonable responses to risks?

o In other words, do all reasonable people have the same degree of risk tolerance?

- Despite the rules obvious shortcomings, courts have generally applied an objective test
regarding what must be disclosed (probably correctly so) and whether the patient would
have undergone the procedure with disclosure.

CHAPTER 14 COMMON LAW STRICT LIABILITY

ABNORMALLY DANGEROUS ACTIVITIES

RYLANDS v. FLETCHER
House of Lords, 1868.

Page 526

Procedural History- Trial Court, Court of Exchequer, held that P had established no cause of
action. The Appellate Court, the Court of Exchequer Chamber, overruled and held P was entitled
to damages for the cause of action. D appeals

Facts- Plaintiff, Fletcher, mined a piece of property that he occupies. The Defendant, Rylands, et
al., are the owners of a mill in Ps neighborhood. The defendants hired an engineer and
contractor to build a reservoir to hold water under their land. When they began to fill the
reservoir with water, the water breached the vertical shafts and spewed onto Ps mines, causing
considerable damage.

Issue(s)- Did the Court of Appeal err in ruling that the P had presented a cause of action which
entitled him to damages?

Holding- No. A person who, for his own purposes, brings on his land and collect and keeps there
anything likely to do mischief if it escapes, must keep it in at his peril; and if he does not do so, is
prima facie answerable for all the damage which the natural consequence of its escape.

Rule(s)- A person who, for his own purposes, brings on his land and collect and keeps there
anything likely to do mischief if it escapes, must keep it in at his peril; and if he does not do so, is
prima facie answerable for all the damage which the natural consequence of its escape.

75
Reasoning/Analysis-

Disposition- Affirmed

Strict Liability Notes

- Liability without fault

- Animals, Abnormally Dangerous Activities, Products Liability

- Animals:

o Under English Common law- liability without fault for damages caused by
trespassing animals (livestock)

- American common law-

o fencing out- rancher oriented- if you want to hold your neighbors liable, then you
have to have a fence around your property

o fencing in- farmer oriented if you want to hold your neighbors liable, then you
must fence your animals in

o local variations in same state

o Wild animals- Both English and American common law impose strict liability

o Domestic Animals-

Strict liability if reason to know of vicious propensity

Otherwise must prove negligence

o When animal owner is subject to strict liability, legal cause must still be established,
i.e. the harm must be of the type reasonably foreseeable given the nature of the
animal and plaintiff must be reasonably foreseeable

o In modern applications of the strict liability rule, comparative negligence may be


used as a defense

- Abnormally Dangerous Activities

o One who brings onto his land something which poses risk of harm to others, should
the thing escape from the owners land, is liable without fault. Rylands v. Fletcher

o If the engineers were negligent in the Rylands case, what arguments would the P
make against the landowner?

Non delegable duty

o Rylands as later interpreted and applied:

76
Activities that are abnormally dangerous (abnormal use or activity)

Creates danger to others

Give rise to strict liability for injuries cause by such activities

o Strict liability has not been universally applied by US in abnormally dangerous


activities

o How to determine what is abnormally dangerous?

o Factors to consider in making a determination as to whether activity is abnormally


dangerous: Restatement (2d) 520

High degree of risk of harm

Likelihood that such harm will be great

Effectiveness of reasonable care in avoiding the harm

Common usage (more common= less likely it is to be abnormally dangerous)

Appropriateness of the activity in the place where it occurs

Value to the community

o The kind of harm (and person harmed) must be foreseeable

Is what happened the foreseeable result of this conduct?

If No, then there is no claim

o An activity is abnormally dangerous and therefore subject to strict liability if it


creates a foreseeable and highly significant risk of physical harm even when
reasonable care is exercised and it is not a matter of common usage.
Restatement (3d) can use this standard or the one above for the test

o Always considered an abnormally dangerous activity:

Dynamiting

Pile driving

o Not really looking at the danger to yourself in strict liability. Looking at dangers to
other people in performing a certain activity

- Purpose of strict liability rule

o Person who is not engaging in the activity should not have to bear the loss. It is
more equitable for the person who is engaging in the abnormally dangerous activity
to bear the loss because they are the person that is creating the risk of loss to
others.

77
SEIGLER v. KUHLMAN
Supreme Court of Washington, 1972

Page 530

Procedural History- Trial Court issued directed judgment for defendants. P appealed to Court of
Appeals, which affirmed. P appealed to S.C. of Washington which reverses.

Facts- 17 year old Carol House died in a gasoline explosion after her car encountered a pool of
thousands of gallons of gasoline. On the early evening of November 22, 1967, while driving
home from after school job in Olympia, she was involved in an accident with a gasoline tanker
that was driven by Aaron L. Kuhlman of Pacific Intermountain Express. Kuhlman testified that he
was driving down hill when he felt a jerk and notice in the rearview mirrors that the trailer had
disengaged from his truck. The tank trailer catapulted off the freeway through a chain-link fence
and landed upside down on Capital Lake Drive. There was no evidence of impact with the
vehicle, except for the fact that the left front headlight was broken.

Issue(s)- Did the trial court err in refusing to give an instruction on res ispa loquitor? Should the
trial court and Appellate Court have used strict liability arising as a matter of law from the
circumstances of the event?

Holding-Yes. This is a case of strict liability as the activity that caused the injury is considered
abnormally dangerous. Transporting gasoline as freight carries a high degree of risk; it is a risk
of great harm and injury; and it creates dangers that cannot be eliminated by the exercise of
reasonable care.

Rule(s)- A person who, for his own purposes, brings on his land and collect and keeps there
anything likely to do mischief if it escapes, must keep it in at his peril; and if he does not do so, is
prima facie answerable for all the damage which the natural consequence of its escape.

Restatement of Torts, Section 519-Strict Liability

(1) One who carries on an abnormally dangerous activity is subject to liability for harm to the
person, land or chattels of another resulting from the activity, although he has exercised
the utmost care to prevent such harm.
(2) Such strict liability is limited to the kind of harm, the risk of which makes the activity
abnormally dangerous
Restatement of Torts, Section 520- What Constitutes Abnormally Dangerous Act?

In determining whether activity is abnormally dangerous, the following factors are to be


considered:

(a) Whether the activity involves a high degree of risk of harm to the person, land or chattels
of others;
(b) Whether the gravity of the harm which may result from it is likely to be great;
(c) Whether the risk cannot be eliminated by the exercise of reasonable care;
78
(d) Whether the activity is not a matter of common usage;
(e) Whether the activity is inappropriate to the place where it is carried on; and
(f) The value of the activity to the community.

Reasoning/Analysis-

Disposition- Reversed

79
CHAPTER 15 - PRODUCTS LIABILITY
EMERGENCE OF STRICT TORT LIABILITY

GREENMAN v. YUBA POWER PRODUCTS, INC.


Supreme Court of CA, 1963.

Page 540

Procedural History- Trial Court ruled that there was no evidence that the retailer was negligent or
had breached any express warranty and that the manufacturer was not liable for any breach of
an implied warranty. Trial Court jury returned a verdict of $65,000 for plaintiff against the
manufacturer, but returned a verdict for the retailer against the P. Plaintiff appeals, seeking a
reversal of the judgment in favor of the retailer.

Facts- Plaintiff brought this action for damages against the retailer and manufacturer of
Shopsmith, a combination power tool that could be used as a saw, drill, and wood lathe. His wife
bought P a Shopsmith as a Christmas present in 1955. In 1957, he bought the necessary
attachments needed for a project he was working on. After he had worked on a piece of wood
several times, the attachment flew out of the wood and struck him in the head. 10 and
months later he notified the retailer and manufacturer of his claim for breaches of warranties and
filed a claim for said breaches and negligence.

Plaintiff introduced adequate evidence that his injuries were caused by defective design and
construction of the Shopsmith. His experts testified that inadequate set screws were used to
hold parts of the machine together so that normal vibration caused the tailstock of the lathe to
move away from the piece of wood being turned permitting it to fly out of the lathe.

Issue(s)- Did the trial court err in granting judgment for the plaintiff against the manufacturer in
that Plaintiff did not give it notice of the breach of warranty within a reasonable time, therefore
barring it from action under 1769 of the Civil Code?

Holding- No. Even if Plaintiff did not give timely notice of breach of warranty to the
manufacturer, his cause of action on the representations contained in the brochure
was not barred. The purpose of strict tort liability is to ensure that the costs of the
injuries resulting from defective products are borne by the manufacturers who put
such products on the market rather than the injured person who are powerless in
protecting themselves. To establish the manufacturers liability, it was sufficient that
P proved that he was injured while using the Shopsmith in a way it was intended to be
used as a result of a defect in the design and manufacture, of which P was not aware,
that made the Shopsmith unsafe for its intended use.

80
Rule(s)- A manufacturer is strictly liable in tort when an article he place on the market,
knowing that it is to be used without inspection for defects, proves to have a defect
that causes injury to a human being.

Reasoning/Analysis-

Disposition- Affirmed.

Products Liability Notes

- A specific type of tort law

- Narrow and focused on a specific range of cases

- Refers to civil liability for injuries caused by defective products (humanly processed items,
artificial, or involve human activity to bring to market)

DEFECT

GRAY v. MANITOWOC CO., INC.


United States Court of Appeals, 5th Circuit, 1985.

Page 544

Procedural History- Earnest Gray and his wife Hughlene Gray, Plaintiffs, brought an action
against the Manitowoc Company, for various things arising from personal injuries sustained by
Earnest when he was struck by the boom of a construction crane manufactured by Manitowoc.
His wife joined the action alleging loss of consortium and companionship. They sued under
theories of strict liability, implied warranty, and negligence by asserting that Grays injuries were
caused by a defect in the design of the crane and Manitowoc had failed to provide adequate
warning of the defect.

-Trial Court jury returned a verdict in favor of the Grays, Plaintiffs. Defendant moved for a
j.n.o.v. which was denied. Defendant, Manitowoc, appeals.

Facts- Plaintiff was struck in 2 separate incidents by the butt end of the boom of a Manitowoc
4100W crane while working as an ironworker foreman on a project near Port Gibson, MS. These
incidents occurred while Grays crew was changing sections of the cranes boom and had placed
the boom in a roughly parallel position to the ground. Gray was standing on the left side of the
crane supervising when the crane operator swung the lowered boom in Grays direction, striking
Gray in the back. Testimony at trial established that the operators vision to the left side of the
crane is obscured by the boom when the crane is in boom-down position. Gray contends that
Manitowoc, in addition to already requiring signalcallers on the ground, should have equipped
the crane with mirrors, or closed circuit television cameras.

81
Issue(s)- Did the trial court err in allowing the jury verdict for the plaintiffs to stand? Did the
evidence establish that there was a latent hazard, as required by MS law? Should the j.n.o.v.
have been granted?

Holding- Yes. The record does not support a finding that the blind spot in the 4100W was a
latent hazard. The evidence was overwhelming that the existence of this blind spot was
common knowledge in the construction industry. No reasonable jury could have concluded that
the blind spot of the 4100W was not open and obvious, nor could any reasonable jury have
concluded the 4100W was dangerous to a degree not anticipated by the ordinary consumer of
that product.

Rule(s)- MS version of strict liability for hazardous products says that manufacturers are not
insurers of the products they produce; the existence of a product defect must be established
before recovery may be obtained for a resulting injury.

Consumer Expectation Test of 402A-

Defective Condition- Only applies where the product is, at the time it leaves the sellers hands,
in a condition not contemplated by the ultimate consumer, which will be unreasonably
dangerous to him. Product does not meet the reasonable expectations of the ordinary
consumer as to its safety.

Unreasonably Dangerous- The article sold must be dangerous to an extent beyond that which
would be contemplated by the ordinary consumer who purchase it, with the ordinary knowledge
common to the community as to its characteristics.

Reasoning/Analysis-

Disposition- Reverse and render judgment.

Failure to Warn?

PHILLIPS v. KIMWOOD MACHINE CO.


Supreme Court of Oregon, 1974

Page 548

Procedural History- Plaintiff was injured while feeding a fiberboard into a sanding machine during
the course of his employment with Pope and Talbot. The sanding machine was purchased from
the defendant, Kimwood. Plaintiff brought this action on a products liability theory, contending
the sanding machine was unreasonably dangerous by virtue of defective design. At the
completion of testimony, trial court judge granted Ds motion for a directed verdict. Phillips
appeals.

82
Facts- On the day of the accident, P was engaged in feeding thick sheets of fiberboard into the
sander. Somehow, a thin sheet was mixed with the lot, and inserted into the machine. The
machine regurgitated the thin piece of wood back at P, hitting him in the abdomen and causing
him injury. P contends that the machines design was defective because it did not have safety
devices to protect the person feeding the machine from the regurgitation of wood. There was
evidence presented that established that for a relatively low price, the machines could have
been installed with a safety device to stop these types of injuries. Evidence was also presented
that after the accident, that these safety teeth were inserted on the machines by the
employer.

Issue(s)- Did the trial court err in providing directed motion for Defendant?

Holding- Yes. Although the employer decided to buy the machine without the automatic feeder
from D, a failure to warn can make a product unreasonably dangerous. The D, who knew that
the Ps company intended to feed the machine manually and who had constructive knowledge of
the machines propensity to regurgitate smaller pieces of wood, and who failed to warn the
employer of the problem, is liable.

Rule(s)-Misdesign v. Mismanufacture- The process of determining what is unreasonably


dangerously defective is much easier in the case of mismanufacture than misdesign.

A dangerously defective article would be one by which a reasonable person would not put into
the stream of commerce if he had knowledge of its harmful character.

-Test for what is a dangerously defective article: Whether the seller would be negligent if
he sold the article knowing of the risk involved.

Seller-oriented standard v. User-oriented Standard- These standards are not inconsistent


with one another as they both turn on foreseeable risks. They are two sides of the same
standard. A product is defective and unreasonably dangerous when a reasonable seller would
not sell the product if he knew of the risk involved or if the risks are greater than the reasonable
buyer would expect.

Reasoning/Analysis-

Disposition- Reversed and Remanded

Notes-

- Phillips v. Kimwood Machine Co.

o What is the appropriate test for unreasonably dangerous defect?

o Consumer expectation or Risk/utility (but in hindsight)

83
o On the other hand they like the fact that open and obvious dangers bar recovery
and that the plaintiff has the burden of proof

o Mismanufacture- the product deviates from the original blueprint/specifications (can


be compared to others that are perfectly normal)

o Design defect- built to specification but the design is dangerous and poses a risk of
harm to consumers

o Court tries (unconvincingly) to reconcile the two approaches. Flaw is particularly


apparent in the effect of open and obvious dangers.

o Most courts have moved to a risk/utility based approach for design cases

o Less explicit about manufacturing defect cases, where consumer expectation still is
a significant factor in some sense

o In warnings cases, courts generally adopt a negligence based approach:


(information defect)

Seller/manufacturer must warn about dangers that were known or that were
reasonably knowable (constructive knowledge) at the time the product was
sold

o Phillips bucks this trend by attempting to make warnings cases truly strict liability:

Would a reasonable manufacturer have warned if it had known what is known


at the time of trial?

o Design defect: the concept makes the product unreasonably dangerous

o Manufacturing defect: the produce deviated from the concept (blueprint) in such a
way as to cause the product to be unreasonably dangerous

o Warnings defect: the manufacturer failed to give adequate warning about the
products risks, failed to provide adequate warning about proper use, etc., so that
the product is unreasonably dangerous.

o In manufacturing defects:

Strict liability is generally applied (with negligence being an alternative)

o In design cases: jurisdictions are more divided

Strict liability using consumer expectation

Strict liability using risk/utility in hindsight

Strict liability using negligence:

o Reasonable alternative design available at the time of sale

Reasonable design involves a risk/utility analysis, and involves cost


issues, performance issues, foreseeability of the risk

o Alternative design test for design defect- strict liability- do not look at foreseeability,
look at cost and performance

84
Cost issues-

Performance issues- utility issues- is the new product as efficient as the older
design?

Foreseeability issues- negligence - difference between

Other risks involved- are the new dangers created by creating an alternative
design more dangerous that the only risk?

Value of the products use

Rx drugs- beneficial to the point that they are worth some degree of
risk

Aesthetics

Technology- is it something that is available?

o Consumer expectation test- is this product more dangerous than a reasonable


consumer would expect it to be?

o Most courts have moved to a risk/utlity based approach for design cases

o Less explicit about manufacturing defect cases, where consumer expectation still is
a significant factor in some sense.

o Most courts will use the negligence test

Drug Manufacturers- Duty to Warn of defects?

FELDMAN v. LEDERLE LABORATORIES


Supreme Court of New Jersey, 1984.

Page 554

Procedural History- Trial Court ruled for D. Appellate Division Affirmed. SC of NJ remanded the
cases to the Court of Appeals and it was again affirmed. P appeals yet again on the issue of
strict liability.

Facts- Plaintiff has gray teeth as a result of taking a tetracycline drug, Declomycin. Plaintiffs
father, a physician, administered her the drug when she was an infant to control upper
respiratory and other types of infections. The drug was manufactured by Defendant. Plaintiff
brought suit not because the drug was defective in controlling infections; rather, the suit was
brought for the failure to warn about the drugs side effect, tooth discoloration. P took the meds
from 1960-63. Evidence was sharply contradicted as to whether the D knew of the side effect in
1960, but by 1962, D did know of the risk and amended its labeling. D claimed that it was not
liable because it did not know of the risk at the time its literature and product were
disseminated.

85
Issue(s)- Did the trial court err in ruling that the Doctrine of Strict Products Liability applies drug
manufacturers? Was the medicine prescribed unavoidably unsafe?

Holding- No. Drug manufacturers have a duty to warn of dangers of which they know or should
have known on the basis of reasonably obtainable or available knowledge. The medicine was not
unsafe or unavoidably so. Drugs, like any other product, may contain defects that could have
been avoided by better manufacturing or design. Whether a drug is unavoidable unsafe must be
determined on a case-by-case basis. Prescription drug manufacturers should not get a blanket
immunity from strict liability manufacturing and design defect claims under comment K.

Rule(s)- Drug manufacturers have a duty to warn of dangers of which they know or should have
known on the basis of reasonably obtainable or available knowledge.

-The difference between strict liability and negligence is commonly expressed by stating that in a
strict liability analysis, the defendant is assumed to know of the dangerous propensity of the
product; whereas in a negligence case, the P must prove that the D knew or should have known
of the danger.

-The question in strict liability designdefect and warning cases is whether, assuming the
manufacturer knew of the defect in the product, he acted in a reasonably prudent manner in
marketing the product or in providing the warnings given.

-As to warnings, generally conduct should be measured by knowledge at the time the
manufacturer distributed the product. Did the D know, or should he have known, of the danger,
given the scientific, technological, and other information available when the product was
distributed; or in other words, did he have actual or constructive knowledge?

-In strict liability warning cases, the D should properly bear the burden of proving that the
information was not reasonably available or obtainable and that it therefore lacked actual or
constructive knowledge of the defect.

Reasoning/Analysis-

Disposition- Reverse and Remanded.again

86
Design Defect Claims

POTTER v. CHICAGO PNEUMATIC TOOL CO.


Connecticut Supreme Court, 1997.

Page 561

Procedural History-

Facts- Shipyard workers claim they were injured as a result of using defectively designed
pneumatic hand tools manufactured by Ds.

Issue(s)- Did the trial court err in applying the consumer expectation test to determine whether
liability should be enforced? Should, as defendants urge, the court abandon that standard and
require the P to prove the existence of a reasonable alternative design in order to prevail on a
design defect claim?

Holding- No. The feasible alternative design requirement imposes an undue burden on Ps that
might preclude otherwise valid claims from jury consideration. This would also require the P to
obtain an expert witness, and in CN, it has commonly been held that an expert is not needed if
the jury can infer a defect from the evidence. The availability of a feasible alternative
design is a factor the P may, rather than must, prove in order to establish that a
products risks outweigh its utlity.

Rule(s)-Consumer Expectation Test- Section 402A- A manufacturer is strictly liable for any
condition not contemplated by the ultimate consumer that will be unreasonably dangerous to
the consumer. Comment i of 402A- Unreasonably Dangerous- article sold must be dangerous
to an extent beyond that which contemplated by the ordinary consumer who purchases it, with
the ordinary knowledge common to the community as to its characteristics.

Or

Modified Consumer Expectation Test-Balancing Test that inquires whether a products


risks outweigh its benefits- Under the risk/utility test, the manufacturer bears the burden of
proving that the products utility is not outweighed by its risks in light of various factors such as:

1. usefulness of the product


87
2. the likelihood and severity of the danger posed by the design

3. the feasibility of an alternative design

4. the financial cost of an improved design

5. the ability to reduce the products danger without impairing its usefulness or cost

6. the feasibility of spreading the loss by increasing the products price

-Other jurisdictions apply only a risk/utility test to determine whether a manufacturer is


liable.

Reasoning/Analysis-

Disposition-Affirmed.

Notes-

- If you can prove consumer expectation or reasonable alternative design theory, use either
or.

- HOWEVER, in MS- you have to prove BOTH consumer expectation AND reasonable
alternative design theory.

- State of the art- what is technologically feasible and cost effective or standard custom
of the injury.or

Causation

- Must be proven in products liability

- Cause-in-fact- remains a required element of proof, even in strict liability

- Difficyl cases for plaintiffs:

o Warnings

Rebuttable presumption

o Crashworthy cases

Do car manufacturers have the duty to take into account crashes? Even from
drivers negligence?

Courts typically say yes

o Drug cases- who manufactured?

- Test

o Duty

88
Negligence?

Strict liability?

o Breach

Collapses with causation in a strict liability case

Defect existed when the product left the manufacturers control and there
was a causal relationship between the injury and the defect

If product was changed after it left the manufacturers control, then it is not
the same product, therefore no liability

o Cause in fact

Injury was caused by an unreasonably dangerous product

o Legal cause

o Damages

- Legal cause is very rarely and issue

o Foreseeability is hindsight

o Would this injury have been foreseeable if the defect was known before?

o Plaintiff must be foreseeable but, if hindsight test, danger need not have been
foreseeable (at lease if the use was foreseeable). However, even then, court is likely
to use foreseeability in some way, as in now that we know the risk, is the risk one
that would be considered reasonably foreseeable in the future.

Potter v. Chicago Pneumatic Tool Co

- What is the appropriate test for defect?

o Consumer expectations

o Risk Utility

o Combination?

- Restatement 3d (general approach in the US, but not the strongest- there is a lot of
mixture)

o Unreasonably dangerous when the foreseeable risk of harm could have been
avoided or reduced by adoption of a reasonable alternative design

- Either or approach Restatement 2d

o Consumer expectation- more dangerous than contemplated by ordinary consumer


who purchaser with common knowledge of the community

- Connecticut court:

o Feasible alternative design may be a factor to consider in determining whether a


product is defective, but it is not a requirement

89
o Some products may be unreasonably dangerous even though there is no feasible
alternative design

o A jury may be able to infer a defect in some circumstances without expert


testimony as to alternative design

o Not comfortable with adopting the Restatement 3d

- Causation:

o Cause in in fact remains a required element of proof

o P must prove that the defect existed when the product left the
manufacturers/sellers control

o P must prove that the defect was a cause in fact of the injury

- Difficult cases for Ps:

o Warnings

Rebuttable presumption

o Crashworthiness cases- trying to figure out how much of the injury could have been
avoided had the product been property designed (car airbags, ex. )

o Drug cases- who manufactured?- DES cases- how to decide who manufactured the
drugs?

- Legal Cause

o P must be foreseeable, but if hindsight test, danger need not be foreseeable (at
lease if the use was foreseeable)

o However, even then, court is likely to use foreseeability in some way, as in now
that we know the risk, is the risk one that would be considered reasonably
foreseeable in the future

- By standers:

o Elmore v. American Motors Corp.

Bystanders injured by product defect may bring strict tort liability claims.

Still must pass test of foreseeability

Have to prove causal connection

Legal cause- foreseeability

Cause in fact- but for test

o In most states, strict liability applies to any seller of a defective product, as long as
the seller is in the business of selling the product

Includes manufacturers, wholesalers, and retailers

o A few states apply strict liability only to manufacturers (must be within the
jurisdiction of the court and not bankrupt)
90
- Pure economic loss

o General rule: P may not recover damages in tort for physical damage to the product
itself. (Ordinarily such a claim sounds in warranty or contract)

o If you buy a computer and it blows up, you cant sue for the damage to the
computer. However, if it burns the house down, you can sue for the damage done to
the house. Have to sue under warranty or contracts instead for the damage to the
computer.

o P may recover for physical damage to other property caused by

o Saratoga Fishing

o Note 1- Majority view: cannot recover in tort for damage to the product itsef

o Note 2- tort recovery is allowed for physical damage to other property and
economic loss caused by such physical damage or personal injury

- Defenses based on consumer conduct:

o Daly v. General Motors Corp.

Can contributory negligence of P reduce Ps recovery in a strict liability


products claim?

YES!!!!!!

o Comparative

Torts Review

- Strict liability

o Animals

o Abnormally dangerous activities

(note: structure is the same as strict products liability- what is the standard
that pploes: strict liability or negligence? If negligence (i.e., not abnormally
dangerous), then normal duty-breach-cif-legal cause- damages approach. If
strict liability is the appropriate rule, then that establishes the standard (in
place of duty of reasonable care) and breach simply becomes a matter of
establishing that the standard of strict liability applies to these facts)

o Products liability:

Duty: Determine which theory(ies) apply. Generally rules apply to users and
bystanders (unless determined to be unforeseeable as matter of legal cause)
as plaintiffs. As to defendants, generally manufacturers, wholesalers, retailers

Negligence

91
Strict liability

o Manufacturing

o Design

402-A

Consumer risk expectation

Risk/utility in hindsight

Restatement 3d

92

You might also like